You are on page 1of 73
{for which ef the following? At and expiration At end inspiration = During @ Valsalva manoeuvre Ifthe patient has a trac’ 20stemy When taking @ deep breast mo@e > 2. A30-">ar-old alcoholic man presents with acute severe upper abdominal pain and vorniting. He is admitted to the intensive therapy unit with a diagnosis of severe acute pancreatitis. Forty-eight hours !awr he develops peripheral paraesthesiae and carpopedal spasms. The mest 4. likely underlying metahotic abnormality is: 7 : A. hypercalcemia | B. hypematraer » © hypocatcaemia t D. hypokalaemia E. hyponatraemia : 3, A 50-year-old woman attends her General Practitioner due to a change in appearance. She finds difficulty removing her rings, reports an increase’in shoe size and photographs reveal a z change in her facial appearance. Visual field tests are-performed to direct confrontation, Whi’ of the following dafects is most likely'to be associated; with her preseniation? b n [ 4. “The pressure within the pleura! space is negative with respect to atmospheric pressure. | | | | | A. Binasal hemianopia 8) Bitemporalhemianopia , Homonymous hemianopia Inferior quadrantano; Unilateral loss of vision noo® 4, A 24-year-old man falls off his motorbike, fracturing one arm, both femurs, and rupturing his spleen. He required surgery .7d 20 units of blood. Twenty-four hours’after admission he has passed 350 ml of urine. His blood pressure is 90/50 mmHg and he has low skin turgor. Which of the following best describes the char Jes in the renin-angiotensin system initiated at the juxtaglomerular apparatus in the patient? : Renin Angiotensin Il | Aldosterone concentration | concentration | concentration J + t L 7 ololal>| L 5. A 35-year-old woman presenis to the Emergency Department with a second attack «| frank haematuria. What is the most likely pathology underlying this presenisstion? [ A. Gram negative urinary tract infection B. Interstitial cystitis ©. + Renal cell carcinoma a D. Transitional celi carcinoma of bladder Urinary tuberculosis 60-year-old man presents wiih a short history of pain in the right cheek and right upper teat! Uyxilary situs infection is diagnosed, This sinus is paiticularly prone to infection because or ab vice of cilia on the epithelium lining the sinus closeness of the sinus to the nasal cavity 0 poor blood sur: piy Position of the sinus ostium high on the medial wall relationship of the front teeth to the fluor of the sinus eae moo m VP 9 @ > 7. A 75-year-old insulin-dependent diabetic man has undergone a hemicolectomy. On the first Postoperative day he is ni-by-mouth, on subcutaneous insulin, maintenance IV infusion with N Saline and intravenou8 Tiorphine via patient controlled analgesia. Ho is confused ler a brie convulsion, has slurred speech and weakness of his right side. His pulse is 110 beats/minute, respiratory rate 25 breaths/minite, blood pressure 160/95 mmHg and SpO2 95% on room a ‘The most likely cause of this convulsion and confusion is: i hyponatraemia + inhibition of noradrenaline re-uptake stimulation of 8 opioid receptors » @ bypogiycaomia B c. D. E, _ stimulation of GABA receptors 2&8. A 30-year-old woman presents to the gone:.t surgical clinic with a 1.5 cin cervical lymph nade along the anterior border of sternocleidomastoid muscle.-Clinical examination and rovtine blood yA tests are: unremar‘able. She undergoes an excisional biopsy of the iump as a day 2ase. The A histology report reveals Yacapsuiated infiltrative carcinoma with marked fibrosis and cystic E changes within the lyriph node: What is the most likely primary pathology? EEE A. Anaplastic thyroid carcinoma B. Follicular thyroid carcinoma + C. Malignant lymphoma of the thyroid * D. Medullary thyroid carcinoma oO L E. Papillary thyroid carcinoma id man presents with severe headache, photophobia and neck stiffness. Kamig's siga is positive and a midline lumbar puncture Is performed immediately to detormine ia Cerebrospinal fluid (CSF) pathogen is involved. In performing this procedure, which is “he fist of { the following structures to be pierced by the lumbar puncture needle? | + A. Anterior longitudinal tigament B. Arachinoid mater Dura mater SQ) interspinous tigament E. Posterior longitudinal ligament G2 10. A 26-year-old woman presents with severe headache, photophobia and neck siiffiess. Among ‘other investigations, a lumbar puncture is cafried.out and cerebrospinal fluid is sent for microscopy and culture. Culture shows a growth of Neisseria meningitidis. What is likely to be the predominant cell type in the cerebrospinal fluid? : tsa co A. Basophils : B. Eosinophils C. Lymphocytes Macrophages Dd \ ©) Neutrophils a 11, A 70-year-old woman from @ nursing home prostifts to'the Emergency Department with abdominal pain and vomiting. On examination; stie'is|dehydrated ‘and her abdomen is distended. There is a 3 cm x 4 cm swelling in het right’groin which is non-tender, and there is no cough impulse. At operation,.a femoral hemnia(is fdund. Which of the following lies madial to the neck of the hernia? { pears A. > Femoral artery Femoral nerve Femoral vein : a ee ® Lacunar ligament ectineal ligament 12. 4 26-year-old man presents to the Emergency Department with extensive bleeding from his ri arm alter sustaining a glass injury. On examination there is a 7 em transverse laceration tl across the anterior aspect of his elbow. On exploring the cubital fossa you would expect the brachial artery to be: i 4 i A. anterior tolthe median nerve i I Br. lateral to the biceps tendon . t . CC. lateral to the median nerve ' D. medial to the median nerve £ E, superficial to the bicipital aponeurosis. t | | 13. 52-year-old man is found to have multiple myeloma. What skull X-ray appearances would be characteristic? i | A. Diffuse thickening of the calvarium B. ‘Hair on end’ appearance 5 © ‘Multiple fractures oO Pee Multiple osteolytic lesions Multiple osteosclerotic lesions. 14, 50-year-old woman prese’ ‘s with a history of right upper quadrant pain and jaundice. She reporis that her urine was dark in colour and that her sicols are offensive and difficult to fluch. | Which of the following explains the dar’ urine? L i eee ong ease in conjugated bifirubinuria { B, increase in unconjugated bilirubinuria C. Increase in urea excretion D. increase in urinary urobilinogen ' E, Reduced enterohepatic bile salt circulation > A45-yeerold man presents with 9 Pain due to a prolapsed lumbar i pervertebral disc, The pain, which ic foe quated by coughing and sneezing, radiates to the lateral aspect of the foot, On examination, thers is weakness of the Plantar flexors of the foct, Which nerve rootis mast ikaly te bo itwvoived? 5 A T12- a ou »\ 84 C68 "A S2 18. A62-year-old woman presents to her & Pain. She has lost 8 cm in height over E SEE Adjusted (corredted) calcium {eae E lated glomerular fitra Catee glomerular filration rai / Parathyroid hormone (PTR (j a Which. of the following is the most likely cause? . 1 in 1 che \_Hypoparathyroidismi foot! : Prt “ary hyperoarathyroidtem i A ty: ©. Pseudohypoparethyroidism BK D. Secondary byperparathyroidismn " +o Tertiary hyperparathyroidism, al ¢ 17. A2 t-year-old ‘Man fractures his ulna and radius. He is treated by means of a plaster cast, i ich is removed after four wesks, Wher Pathologica! procoss is most icely to have ofetica Ly the immobilised muscles? A: fil Aptasia Le} + @& Atrophy C. Hyperplasia 5 D. Hypertrophy : i E. Neoplasia A. Left atrium le B. Left lobe of the liver Left ventricle i D. Right atrium +E) Right ventricle 8 An 80-year-old man dislocates his shoulde- It is reduced in the Emergency D Troe nlite Weeks later he is unable to acth lepartment. At Vvely initiate abduction of his aren, most like: jamaged? : 1. Which muscle is t A. Clavicular he Infraspinatus Supraspinatus Teres major Trapevine l| : "ad 2 pectoralis major { I 20. An 8-year-old boy is admitted for assessment of rectal blood loss, His mother describes this as bright red blood in the toilet pan. He has'no nai an defadcation. ‘There is no family history. What is the most likely cause? i A. Adenomatous polyp Familial adenomatous polyposis 2 (C/ suvenile polyp Metaplastic polyp , Peutz-Jegher's syndrome 21. A.48-year-old woman presents with abdominal pain and five to six episades.of bloody diarthoea each day, She also complains of pain in thé knées and elbows and recent viguiat problems. What is the mostlikely diagnosis? :- | A, Collagenous colitis ote B. Diverticulitis ot & Infestive colitis ‘testinal tuberculosis ote Le Ulcerative cetitis 22. A 86-year-old man presents with acute epigastric pain and vorriting. On examination he has guarding in the upper abdomen. Investigations reveat: S900 U/L 61 TUL __ Serum amylase _ (Serum ACT (Alanine transferase) Alkaline phosphatase Albumin > EF 98 WIL bq 38 git, Gamma GT |, * 761A t ~ Corrected calcium 3.30 mmol Bitirubin fv 15 moll. { mmol ~Tiigiycerides ‘Which of the following is the most likely aetiology of this condition? Alcohol Choledocholithiasis, ) Hyperparathyroidism 6 rt ‘ ? ‘BY Hypertriglyceridaemia i Mumps 23. 30-year-old man is adruitted to the intensive care unit with an isolated severe head injury. ACT “can shows multiple intracerebral bleeds but 0 midline shift. He is intubated and ventilated. His pupils are dilated and react sluggishiy to light. His heart rate is 50:beats/minute, blood pressure 176, 110 mmHg and his respiratory cate is set at 10 breaths/minute. The rising blood pressure is likely to be caused by: ‘A, aortic and carotid baroreceptor stimulation B. cortisol stimulation CC, renin-angiotensin stimulation D. sympathetic stimulation related {o blood foss E. sympathetic stimutati related to intracranial pressure 24. 4 42-year-old multiparous woman is admitled to the Emergency Department dus to pelvic. discomfort. The duly gynaer ‘ogist diagnoses uterine-prolapse. Which anatomical siructure gives significant direct support to the uterus? : Cervical ligaments *ssometrial part of the broad ligament ‘Mesosalpingiat part of the broad ligament Round ligaments Oo Transverse perineal muscles moom> 25. An 80-year-old man is admitted to the surgical admission unit with central abdominal palh. His blood pressure is 100/60 mmHg and his pulse is 140 beats/minute with a respiratory rate of 2 breaths/minute. He is known to have severe chrortic obstructive pulmonary disease | (COPD) and has been @ heavy smoker all hs life. On examination he has a rigid abdomen. Arterial blood gases show: * ae { pH | p02 _)ApCO; [Bicarbonate | Base excess] | Finding | ¥7.24 | YowPa [45.3 kPas\b 17 mmol. 3. | Normal | 7.36-7.45 | 11.9-133 | 47-60 |" 22.26 | -2lo+2 What is the most likely diagnosis? A. Hypovolaemic shock complicated by COPD B. Metabolic acidosis due to COPD + C. Metabolic acidosis due to peritonitis D. — Respiratory-acidosis due to COPD E. Respiratory acidosis due to peritonitis 28. In describing the sino-atrial nade, which one of the following statements is correct? ea Itis part of the somatic nervous system é& Itis usually supplied by the left covonary artery C. It ies in the wall of the left atrium uy ©. 1.0s inthe. wall of the right ventricie +E, Itreceives fib + derived from the vagus nerves $27. A fit 21-year-old man is admitted with an acute abdomen, subsequently diagnosed as gastro- : enteritis. As part of the host immune response, which of the following immunoglobulins ard | cells are correctly paired? A. IgA-macrophage ' | B. _ IgD-small bowel epithelium L © f 28. 45-year-old man presents with fever and pain in his right loin and groin. A soft swelling was ky noted in his femoral triangle. Diagnosis of a psoas abscess was made. Which of the following aa statements is most accurate regarding psoas major? | arises from the lateral borders of the bodies of T12 to LS Itextends the hip it inserts into the greater trochanter of the femur Itis innervated from 712 and Lt hip joint Qo mo6e> It passes posterior to the capsule of t | 8 29. Which one of ne following musoles is aii extensor of the hip? A... Adductor tongys B.” Gracilis : liiopsoas Pectineus Semitendinosus 30. A tall, thin, 25-year-old woman presents to the surgical clinic with a complaint of swelling in the front of her neck for the-last four months. On clinical examination she has a swelling in the left lobe of the thyroid and multiple neuromatous:lesions within the oral cavity, Her blood pressure is 220/120 mmHg, Laboratory. investigations show that her calcium and electrolytes are normal and serum calcitonin and urinary vanilylimandelic acid levels are elevated. ‘Weich of the following is the most likely cause/of the hyperiensidn? ‘A. Con's syndrome B. Cushing's disease Sx Essential hypertension «@) Phaeochromocytoma : E. | Renal artery stenosis eeae 31, A 65-year-old man presents with haematurie and left loin pain. Computerised tomography derr-nstrates a left renal tumour. He undergoes a left radical nephrectomy. Where does the left renal artery ie? i ‘A. Anterior to the left gonadal vein B. ~ Anterior to the lefl renal vein : &. Anterior to the splenic vein . » @ Posterior to the left renal vein E. Superior to the superior mesenteric artery 32. Within the posterior triangle, which nerve is at particular risk of damage during surgery? A. Auricular branch of the facial B. Glossopharyngeal Cc. Phrenic i D.. Recurrent laryngeal yE ) Spinal accessory Qo 33. The right and loft pulmonary arteries are derived from which of the following embryological ‘aortic arches? A. Second aortic arch B. Third aortic arch Fourth aortic arch ifth aortic arch Sixth aortic arch © 4. A47-year-old man presents to the Emergency Department six hours after a climbing accident. fy initial assessment suggests that he had lost 4 L of blood. His blood pressure is 105/70 mmlig and his pulse rate is 88 beats/minute. He is catheterised and his bladder contains 120 ml of urine. Activation of which of the following transport systems best describes how aldosterone feads to maintenance of the intravascular volume and oliguria in this men? A. Na’/glucose in the proximal tubule B. _Na‘/H" in the descending loop of Henle C. _Na‘/K’ in the ascending loop of Henle D. E Na*/CF in the distal tubule Na’/k* in the collecting ducts Qo f : 35. During arch'aortography, the vertebral artery would be seen to arise directly from which of the a following? : : A. Ar. ofthe acta B. Brachiocephalic tery | €. Common carsiid artery Internal carotid artery | 2 CE) Subclavian artery 36. A 65-year-old man remains.on the high dependency unit following a gastreatorny three days | previously. His urine output has averaged 80ni/hour forthe ldst 24 nours. He has one dry Bbdominal drain and no nasogastric losses: His urea and:electrolytes are normal. He is to stay 7 nif by mouth for a further 24 hours. Which of the following fluid regimes is most appropriate! to | his electrolyte and water requirements over the next 24 hours? eee i : A. 1 Lof0.9 %’saline plus 1.5L of 4 % dextrose/0.18 % saline } 2 B 1Lof0.9 % saline plus 1.5 L of § % dextrose : t C. 4 Lof0.9 % saline plus 1.5 L of Hartman's solution D. 2b of0.9% saline 06 | E, 2Lof4 % dexirose/0,18 % saline | 37. A3-year-old boy is admitted to hospital with severe vomiting. Radiogrephic examination 7 reveals that he is suffering from annular pancreas. Whicit of these structures is constricted? | First part of duodenum t . Second part of duodenum Third part of duodenum Provimal jejunum ‘ Pyiutus of stonnach _ 38. A 20-year-old man with a severe head injury is being ventilated using intermittent positive pressure. Yvhich of the following is a physiolegical consequence of this? 7 A. Decreased extracellular fluid volume - @ Decreased cardiac preload Increase in intrathoracic blood volume 0 D. . Increased cardiac preload E. increased cardiac stroke volume 39. 36-year-old man presents with a two-month history of low back pain radiating to his left leg. ‘After clinical examination he is referred for: an MRI scan. This shows a prolapsed . intervertebral disc. The clinical signs are consistént:with pressure on S1 roots. Which ‘combination of clinical signs is most likely? A. ‘Weak ankle plantarflexion; altered sensation on the dorsum of the ‘oot, riormal ankle jerk reflex. (1B) Weak ankle plantarfexion; altered sensation on the sole of the foot, toss of ankle jerk * etiex. 4 C. Weak hallux dorsiflexion; altered sensation on the anterior surface of the leg; !oss-of ankle jerk reflex. 5 D... Weak haliux dorsifiex.un; altered sensation.on the dorsum of the foct; normal ankle jerk reflex. i E. Weak hallux dorsiflexion; alteres sensation on the sole of the foot; loss of knee jerk reflex. t cystectomy: The gallbladder contains multiple dark, is an increased bloc 40, A 55:yedr-old woman undergoes a cholé iregular small stones. The most likely cause of tus {ype of gallstone { evel of: 4 bile acids calcium 8 cholesterol conjugated bilirubin unconjugated bilirubin nous thrombosis. His full blood count shows: ‘a white cell count of 8 x 10°%L (riormat (0°). This 41. A 85-year-old man presents with a deep ver a haemoglobin of 18.3 gidL. (normal 13.5-17.5), 3,9-40.0 » 10°) and a platelet count of 200 x 10°/L (normal 150-400 x 1 haematological picture is most likely to be associated with: bronchial carcinoma pancreatic carcinoma prostatic carcinoma renal cell carcinoma transitional cell carcinoma of the kidney moop> year-old has had pancolis for the last five years. Which one of nase complications is he likely to develop? 5 Adthritis. Cholangitis Iritis -“ Polyarteritis b-~ 98 Toxic megacolon ~~ lectern {i ‘Asurgeon makes @ ptannenstiel incision for access tothe pelvic organs. He incisos the | odomiral wall down to and “through the rectus sheath He etracts the repius abdomin's aheclos laterally from the midline fo expose the: oO Lj A. linea elba ‘8. _ peritoneum f C. _ posterior rectus sheath ie [ /= 5; trensversalis fos a Z transversus abdorinis muscle a, A25-year-old man undergoes splenectomy ‘ollowing blunt abdominal trauma. Three days se, postoperatively op nous bood tasting is likely € reveal {., a, high platelet count B, high red cell count f C,_ low platelet count a Dp. ow white cell count E, lowed cat count 1G. MAG-year-old worman is SECN ip the pre-admission clinic she had a myocardial infarction |two yyears ago. On examination she ig in atvial fibrillation ‘and a recent echocardiogram sshowsithat | gre has a dilated left Montrigio, She is on warfarin tnd her INR (\ntesnationat Normalised Ratio) ir 6.4. Warfarin inhibits which of the following? A ‘Antithrombin B. Kallikrein cc. Plasminogen = @ Prothrombin —E. von Willebrand's factor ag, ATS-year-old man js admitted for elective repair of an ‘abdominal aortic aneurysr™ Following H iis below 90 MMH and during resuscitation he 18 eration his systolic blood pressure fal + glen an infusion of fluids sThore is a choice of colloid OF raystalloid. Which of the following best describes why the intensive therapy unit registrar decides to administer @ colloid? Galloids are not freely siterable aoross sem-permedb|o membranes Colioids can prevent pulmona’y serena by inducing fuid flow Ou ‘of the interstitial Space ©, Colloids decrease the ranseapiliary oncatic Pressure gradient y Colds expand the plasma VOIUTS ty 200 mi foreach litre ised, D. ©. Colloids migrate from the {intravascular Space develops septicaemia folowing surgery for perforated acute appendicitis. igas analysis reves: AT, A35-yeerold man Ho is nypotansive. Arterial blood Whatis the most likely explanation for thse readings? A. © rpensatert metabolic acidosis g. _ Compensated respiratory acidosis we Gar iixed metabolic and respiratory acidosis D E Uncompensated metabolic wcidosts Uncompensated respiratory 2 Josis 8 | c 48. 20-year-old man presents to the Emergency Department with a stab injury to the.thenar eminence, On examination he is found to ftave-@ 2'cm long laceration with loss of sensation in tHe thumb and index finger and weakness of the thenar muscles. Which of the following structures is most likely to have been injured? A. Anterior interosseous nerve B, Recurrent branch of median nerve ©) Sensory and motor branches of median nerve D. Sensory and motor branches of radiat nerve 0 —, Sensory and motor branches of ulnar nerve 49. A 65-year-old man presents with vomiting and weight logs. Qn examination he had 2 palpatle epigastric mass. The transpyloric plane lies at which veriebral ieve!? : A. Tit BL 712 % ui LZ A 30-year-cid motorcyclist is brought to the Emergency Department after a road traffic accident. He has an open fracture of the right femur. On arrival he is tachypnosic and} confused, with cold and clammy skin, Which of the following physiolagica! changes is fnost likely to ecour? i | A. Decreased reabsorption of water from the collecting tubules i * By Decreased serum bicarbonate levet 7 | C! Increased pH of blood 96 i D. Increased secretion of sodium in the urine i E. increased synthesis of glycogen in the liver 1 51. In an emergency situation what is the most appropriate surgical method of obtaining an airway? A. Cutting and retracting the cricothyroid muscle B. Cutting the thyrohyold membrane C. ‘Dividing the thyroid isthmus: B. Entering the trachea at the C7 level ‘cing the cricothyroid membrane g 52. ATS-year-old man presents wiih her stomegaly and ascites. Computerised tomograpliy shows evidence of post hepatic portal hypertension, The inferior vena cava passes through the ..aphragrr. at which vertebral ievel? 8 To. T10 TH T42 moom> 53. A 20-year-old womian’presents acutely with abdominal pain. imaging reveals a right ovarian | cystic lesion which is excised. jigtotogical examination shows 2.C¥S! lined by keratinising stratified squamous epithelium. ‘Areas of fat, muscle, tayroid end neural tlesus are seen in the Syal. What is the appropriate pathological designation for this lestan’? A Adenocarcinoma _'B,_ Gystadenoma { cc. Dysgerminoma ‘ D. Squamous cell carcinoma « , Teratome i | gd. A 25-year-old woman who Is We months postpartum, presents wih a 60! moss in the left te 7 reek she is pyrexia wih a reised wi rant ane razed C-reactive Prowin revel 1 [ brea a of tetational masts with abscess © “un 3 hatte flowing isthe mos} Hels | ‘causative organism? ih ; | a. Aeromonas hydrophila 4 : B. -Escherichia cof \ 2. “Streptococcus miller! : | { DB. Lactobacillus casell : 1). Stapnylococcus aureus 1 is admitted with severe acute pancreatitis. Three days after poe fs 7.5 KPa) and contused ‘Chest X-ray shows a uniform the most likely clinical diagnosis? t 55. A 62-year-old alcoholic man» ‘edmission he bacomes Tiypoxte r opacification of both tung Yialds. Which of the following Is LA. ‘Acute left ventricular failure iE adult respiratory distress syndrome ' -@ t © Bilateral bronchopneumonie D. _ Bilatoral jovar pneumonia ' E. Massive pulmonary ‘embolism U «50 the Emergency Department wih 2 evar fiuctuant swellingtt ECG shows tial? 56, AS56-year-oid worran Press ineeet bite. She is anxious, tacyca jhe site of a recent it wrdic, and pyrexial. AP | fibration. She is noted to have a goitre. The swaling atthe site ef tine bite requires surgical Painage. Which of the following lasso ty gsug would be most appropriate aS part of her i pre-o, ative prevaration for surgery? A Alphi 4 adrenoceptor agonist B. Alpha-2 adrenoceptor agonist. f ©. Alpha adrenoceptor blocker 1D. Beta adrenoceptor, agonist 0 * & Bela adrenoceptor blacker 57. Anulliparous 20 with 2 recurrent painful red area in tne areola. She i gives a history of smoking ‘garetts. The lesion Is oxcived ‘and the histology shows resus ducts. Microbiotagical culture does not grow any Squamous metapiasia of jactifer ‘organism. Which of the following is the most likely diagnosis? year-old wornan prosents s { A, Breast absoes B, Carcinoma of the breast : ff Fibroadename, | D. Galactorrhea 7 ~ —E, , Subareolar abscess 6 4 afebrile, naust morphine via @ » 5 small intestine. patient-controlled analgesia pum round infection, By what process would A. Apoptosis B, Autophagy Cc. Metaplasia the Emergency Departmer Thorax, A chest drain js inserted into the rtnore is haemorrhare into the drainage ote, Which st likely cause of this acuie haemorrhage? 72. A22-year-old a large pneumo rmid-axiliary line. structures is the mos intercostal artery Left pericaraiophrenic artery Lingula of the jung. 8 7 Right ventricle of the heart a A. B G. D. £. Spleen "ES / /A 65-year-old woman collapses after a total hip replacement. A pulmonary embolism is fee fi Suspected. Which of the following electrocardiogram changes would Support thie diagnosis? A. Dominant wave inv Lett axis deviation Opeminen Ee waur B. C. Left bundle branch block a a bid 1 7 G Leftbundlebranch tock Bau cel haired Tuan a Loe TL E. ( B STetevation in Vi-v3 eo + ale iw ls T wave inversion in V1-V3 on! aad Vow dy Carelel wrth Neto in Vi-vs Rt Bundt Brags, ; n @ lew ompocack bileckay’ “ 1) 4 4.40-year-old woman had the t..erior lobe of the pituitary removed Because of a tumour. Without postoperative supplements, vihich of the following could occur? : lure to proditce adequate amounts of thyroxine Fail. 2 to prod: 2e parathyroid hormone in response fo hypocalcaemia Failure to secrete @lecholamines in response to stress Failure to secréto insulin in hyperglycaemia Inability to concentrate urine in response to weter deprivation 75, AS8-year-oid man i end-stage renal fallure resulting from polycystic kidney disease receives i a cadaveric renal transplant. Good renal function is established but four weeks later [| £ deteriorates, the serum creatinine rising by 25%. Which ofthe following processes is a a | likely to Be responsible for this deterioration? B-céll mediated rejection Ciréulating immune complex disease IgGiantibody mediated fejection a | Post-transplant lymphoproliferative disorder ‘T-cell mediated rejection In order to expose the right exillary artery, a transverse skin incision is typically made below the clavicle from a point just lateral to the sternal end of the zlevicle to the deliopectoral groove. Which of the following structures would be ericountered in the dissection down to the vessel? 3 Lateral thoracie artery Phrenic nerve Suprascapular artery Thoracic duct Thoraco-acromial artery a 77. A4-year-old boy presents to the Emergency Department with a two-dayjhistory of headache, . vomiting and drowsiness. 4 CT scan reveals dilatation of both his lateral ventricias and his third cerebral ventricle. His fourth ventricle was of normei size. It is suspected that he has an obstruction to his cerebrospinal fluid flow. At which of the following sites is the obstruction most likely to be? “MOON Gerebral aqueduct (of Sylvius) Infundibular recess Interventricular foramen (of Monro) Lateral foramen of fourth ventricle (faraman of Luschka) Median foramen of fourth ventricle (foramen of Magendie) 8. c. O, E ie es, Maen] / / 78. A 12-year-old boy presents to the Emergeiicy Department two hours after helping his father cul the grass. He complains of rhinorrhoea, itchy eyes, sneezing and a blocked nose. He is apyrexial with'a haemoglobin of 12.2 gidl. and white blgod cell count of 6.8 * 10° with a raised eosinophil count. Chest X-ray is clear. Which Inimunoglobulin is most likely to cause this ceaction? : ; 8 79, A 35-year-old woman undergoes gastric bypass sutgery for, morbid obesity. At a subesequent su ‘ical clinic reviow she complains of dizziness, sweating, palpitation and collapsing episodes after-bia meals without any vorniting or pain. She is otherwise well and hasjno- . medical problems. What is the most likely. explanation for, her symptoms? Weta A. _ Operative denervation of stomach i : B. Release of gastrin ‘ f2 ; C. Release of glucagon i A sor : . © Release of insulin. ~~? Cs an) we Vasavagal syncope ye nei 80. A 56-year-old man with chronic emphysema is on the high dependency unit (HDU), tan days after antorior resection. He has developed acute shcriness of breathi and hypotension: Which of the following landmarks would be the most appropriate to use to obtain a,femoral aerial blood gas sample? | i A point midway between the anteriar superior iliac spine and the puble tubercle. 4, ‘A point midway between the anterior superior iac'spire’and the pubic sy f A B. C. Apoint midway between the greater trochanter and the pubic ‘symphysis D. E ‘point 1 cm inferior and 4 cm lateral to the pubic tubercle Ene ‘Apoint 2 em lateral to the midway point between the anterior supetior iliac sein and the: pubic symphysis a | 81. A 70-year-old man complains of persistent numbness since.an inguinal hemia repair one year previously. The numbnes. -affects the top of the scrotum, root of penis, and a small area below the medial part of the inguinal ligament. Which nerve is likely to have been damaged? : : A. Femoral branch’of the genitofemoral nerve Eee B. Genital ranch of the genitofemoral nerve C. _llichypoga~ic nerve a ‘D) _Ilicinguiral nerve E. _Mediat cutaneous branch of tne femoral nerve 82. A 12-year-old girl has a diastolic murmur. It is maximally audible in the second left intercostal space. Pathology of which’structure Is the most likely cause? A. , Aortic valve * :B." Ducts arteriosus Cc. Mitral valve D. Pulmonary valve E. Tricuspid valve o f| "A 34-year-old pregnant woman develops a swollen feg, Her mother and maternal aunt also had similar propiemis during their pregnancies. Which of ths following tests is likely to be ive? ‘Anuendomysial antibodies ‘Antimitochondriu. antibodies: ‘Antinuclear antibodies Antiphospholipid antibodies ‘Antithyroglobutin antibodies Oo 4. The “ight or fight, response produces a release of epinephrine {adrenaline). What is the primary metabolic effect of epinephrine? A. Alanine’shuntactivation Cortisol release Glycolysis “Tachycardia Vasoconstriction o 5, A'58-year-olc-man underwent an emergency appendicectamy. Which of the following physloiogical parameters are cnsistent wih sepsis? : | Heart Rate | Systemic “Vincuiar Resistance | Cardiac output A J ; pra t 8 EEE 4 a Toy : L i tt 0 D t [ L E EE ee L | a 80-year-old man with a 30-year history of pancoliis undergoes surveillance colonoscopy. which reveals a plaque-like jesion in the descending colon. Biopsy reveals a pre-malignant |< Ghange. Wht is the name of this pre-malignant change? Anapiasia H pEree Dysplasia Hyperplasia Metaplasia 8 Neoplasia my for a bladder carcinoma, During surgery, the 87. A 60-year-old man undergoes cystector {the bladder do the ureters pierce the bladder wall? ureters are identified. On which region of Anterior surface Apex Lateral surfaces Neck Posterior surface Oo mooeE he intensive care unit nas a tracheostomy performed via the secon, 28. ATw-year-old patientiint rvening structure is most likely to require third and fourth t cheal rings. Which inter 4 transaction? A. Anterior jugular vein [ye 74 _- B. Inferior thyreid veins C. _ Sternothyroid muscle 7 D. Thymus: f + &\ Thyroid isthinus 2 5 : a ; tl assaulted and sustains a stab injury to the left groin. The wound ts dressing applied. Six months later the patient returns with a pulfatle ry. The most probable lesion present is: | i 89, A 23-year-old man is cleaned and a simple swelling at the site of the injur A. abscess . + 8) false aneurysm of the femoral artery, : 8 7 C. lymphocoele | 1D. mycotic aneurysm of the femoral artery : —, saphena varix \ ; 90. A 72-year-old me~ with a body inass index of 18.4 has not eaten for four days following the j aoe Tuenocercinoma from his descerding colon. His urea is found to be 42 irr yL | {normal 32-7.) and ereatinine 246 uimolR. (normal 36-110), A blood gas profile is ordered, ‘ Tho most likely set of results would be: i I [eso, ' 3.8 kPa out_| mee 3 side of the face by @ ball. There ‘are no broken bones Eut | 91. A 16-year-old boy is hit on the eft he face below the eye. Which nerve has most likely b2en A the boy complains of numbness of t compromised? Abcuicens nerve. Facial nerve pharyngeal nerve orbital nerve Qa Zygamatic neve 74-year-old man, who has uridergone emergency major abdominal surgery two days previously, is noted to be confused. He has been on furosemide for mil heart failure. The é plasma sodium is 122 mmol/L. Inspection of the fluid chart shows that he has been written ug for iL, forr-hourly intravenous 5% glucose infusions. What is the most likely cause for the, hyponatraemia? . ‘A. An ACTH (Adrenacorticotrophic hormone) response to surgery 2 (8 Excessive sodium-free intravenous fiuld administration CC. Ostnotic effect of hyperglycaemia induced by glucose infusions 8 D. Syndrome of inappropriate antidiuretic hormone. E, Use of loop diuretic in tho long term Go rr The Cori cycle is important in lactate metabolism in the septic:surgical patient. It is used to | fl desctibe a pathway in which glucose is metabolised anaerobically to lactate in one tissue and Y the lactate is converted back to glucose in another. Which one of the follewing relies on this a cycle tormeet aif ofits energy needs? : ry A: Exythrooyte Bs @ Hepatocyte | . i Leéukooyte i ete a E, Preumooyte I 94, After rec.wving an haramuscular injection in the buttock, a 25-year-old man comptsins of inability to evert his fo. . Which nerve is most likely to have been injured? ; ~ -A. Common peroneal (fibular) component of sciatic g B. Inferior gluteal ©. Pudendal D, Superior gluteal - E. Tibial component of sciatic 95. 73-year-old than with a history of fregutar bowel movements presents with dysuria, pneumaturia and an Escherichia coll urinary tract infection. CT scans show 2 mass involving | fhe sigmoid colon and the bladder. What is the commonest cause of this presentation? ‘A. Adenocarcinoma of the sigmoid caton | + (B) Colonic diverticular disease | C. Crohn's disease 0 1D Transitional cell carcinoma of the bladder E, Ulcerative colitis 8 ~yoman is in the recovery area and receives 28% oxygen by mask. Biood gag ___| yp “TA pce; Finding | V7.2 {770.0 kPa Normal |7.95-7.45 | 47-60 Reduced sensitivity of which receptors is most likely to be responsible for this blood gas picture? q A. Adrenergic receptors B. Baro receptors * (G. Central chemo-receptors 0 D. Jreceptors E. _ Lung stretch-receptors \ 97. A 76-year-old woman falls.and sustains en inter-trochanteric fractured neck of femur. Following operative fixation her recovery is prolonged and she is discharged to a rehabilitation | Unit. Six weeks after her otiginal operation she is readmitted. She is drowsy, hypotensive | and bradycardic, An ECG shows low voltage complexesand a prolonged QT interval Under-activity of which of the fotfowing giands gives the best explanation of lie atin { picture? me A. Adrenal } ry » Bo Pancreas | Bye f ‘© Parathyroid j | D. Pituitary. 8 Tyrie a i 98. A 36-year-old man falls on his outstretched right hand. Examination reveals tenderhess'in the anatomical snuff box. Which one of the following tendons form 2 boundary of the anatomical snuff box? ‘ A. Abduetor pollicis brevis ; B: Extensor carpi radialis brevis { C. Extensor carpi radialis longus a D. Extensor indicis + E> Extensor pollicis iongus 99. A 65-year-old woman with metastatic breast cancer is admitted to hospital acutely unwell, with nausea and vomiting. Her vital signs are within normal limits. What | abnormality is most likely to contribute to her clinical condition? ° tA? Hypercatcaemia j B. Hyperkalaemia & C. hypocalcaemia 0 D. Hypokalaemia | é. yponairacmia . A4-year-old boy presents to the Emergency Department with a two-day history of fever, difficulty walking and is unable to. weight bear on the right leg. He has been on oral Amoxycillin 250 mg three times a day for a citest infection over the last five days. He is irritable with @ temperature of 39.4°C. He does not allow examination and keeps his right nip flexed and abducted. Blood tests are: white blood cell count (WBC) 1.3 10%dL, C-Roaetive protein (CRP) 146, haemoglobin 11.3 g/dl. What is the most likely diagnosis? ‘A. Acute avascular necrosis of hip . B. Dislocation of hip C. Perthes disease a *«D, Septic arthritis. E, Slipped upper femoral epiphysis, 101. A 35-year-old woman presents with recurrent peptic ulceration. She is on proton pump inhibitors and previously received Helicobacter pylori eradication therapy three months ago. Which of the following is likely to be raised on venous bloodtesting? A. Cholecystokinin ; “Bo Gastrin ; : C. Histamine | | BD. Pancreozymin -O 1 E. Secretin | l i f 102. A 56-year-old main presents with back ache. Neurological examination reveals lack of I extension of the feft great toe. Which of the following spinal cotd segmenis are likely:to be involved? i I A. (112813 i B. L2,L3&L4 ~ C1415 881 a8 { D. $1, $2,883 E. $3, S4&S5 l 193. "A 34-year-old man is seen in the Emergency Department with a fibular fracture following football match. On examination he is noted to have loss of foot eversion, Which area of skin . ( ' should be examined to confir. . loss of the cutaneous distribution of the affected nerve? i A. Along the fateral aspect ofthe foot | B. Along the medial aspect of the foot * GC. C the dors! surface of the foot 7 D. Onthe plantar -“wface of the foot + (3 E. Overthe hee! 404. A 68-year-old man with insulin-dependent diabetes presents to the preadmission clinic prior { to surgery for rectal carcinoma. His creatinine is found to be 590 mol/L. He is referred to a renal unit where a measure of his glomerular filtzaiion rate (GFR) is made. Which of the following best describes why inulin could be used to measure his GFR? \ ib “A. Itis filtered and not reabsorbed B. itis filtered and reabsorbed , ©. Itis filtered, secreted and reabsorbed QO l D. tis filtered, secreted but not reabsorbed E, _Itis not filtered but is secreted and not reabsorbed | 24 105. Sasel of right sided hemiparesis. A CT scan demonstrates 4.om > 4 cm lesion with central necrosis in the left frontal lobe. Itis seen ‘extending across the midline along the corpus oefosum, There is extensive oedema around the lesion. Which of the following is the most likely diagnosis of this lesion? . Cental neurocytoma 7 B, Ependy:.oma . * CG, Glioblestoma 2 D. Meningioma E, Oligodendroglioma 408. A 65-year-old woman presents with 2 2.5 em diameter Meee in the upper outer quadrant of the lek breast with associated axillary lymphadenopathy ‘A core biopsy is taken which , confirms the presence of carcinoma. Which of the folowing types of carcinoma is this most likely to be? « G) Invasive ductal carcinoma L ‘B. — Invasive lobular carcinoma C. Medullary carcinoma 1D. Mucinouss carcinoma , Tubular carcinoma 407, A 26-year-old man is admitted to the mergency Depat fractures. He is clinically shocked. Which is the ‘structure responsible for response to @ fal in systemic arterial blood pressure? Adenohypophysis Barorecepior G. Chemoreceptor ~ — -Kidney E, _ Neurohyr-physis 408. Vitamin K is required for norm: ‘boul the effects and availabilty of vitamin K? Affects platelet function Causes bleeding if taken in excess Is provided only by fresh food 1s depleted by broad spectrum oral antibiotics 409. in septic shock, norepinephrine (noradrenaline) is fesistance. This action is a result of the stimulation ‘of which ~ AN at BO cB Df 8 Eb 'A 60-year-old man presents with a one-month histony of headache, confusion and-recent ‘| blood clotting. Which one of the following statements is true 8 Vitarnin K absorption is affectad by respotion of the terminal il used to increase the systemic vascular i os iment with multiple peripticrat the first homeostatic". fin. of the following receptors? 440. A 10-year-old child presents with progressive facial weakness and a squint. On examination there is a lower motor neurone facial weakness and failure of abduction of one eye. An MRE scan shows a small, intrinsic mass lesion within the central nervpus system. Where is this: 4 i } | most likely to be located? | Seneee j i L/ A. The cerebellar vermis, | B. The cerebellar tonsil C. The medulle L D.., The midbrain : /E.- The pons a mic heart disease, has an abdomino-perineal ible, Two days later he Is clammy, has cool f 80/60 mmlg. His temperature is normal. n 111. A 65-year-old man, with a history of ischaer | excision of the rectum after which he is, stal peripheries, a tachycardia and a blood pressure of What is the most likely cause? 5 + Cardiogenic shock Hypovolaemic shock Neurogenic shock 2 Septic shock FI MoO D> tianer with a four-month history of feeling rgement. Investigations reveal: 412. A55-year-old'man presenis to his General Prat tired. The General Practitioner notes breast ent : Which of the following is the most likely cause? { A. Primary hyperthyroidism ae Primary hypothyroidism Secondary hyperthyroidism 98 Secondary hypothyroidism Sick euthyroid synarome 443. A40-year-old woman presents with a parotid tumour, A biopsy reveals extensive perineural invasion. Which is the most likely pathology? A. Acinic cell carcinoma B. Adenocarcinoma + G+ Adenoid cystic carcinoma D. Lymphoma E, Pleomorphic salivary adenoma : 444. “A 50-year-old man presents with malaise, abdominal pain, weight loss, fever and myalijia. Polyarteritis nodosa is ase tiated with: all of the following except: & : ® “way anenaaaom ) ‘ivedo reticularis CC. -mononeuritis multiplex Q : D. nalifold infarats « E. palpable ppure. : 26 445, A 30-year-old woman undergoes subtotal thyroidectomy. Five days fater the wound appea's rad and inflamed. ‘Which of the following 'S the most likely causative organism? A. Haemophilus influenzae a 8. Proteus-mirebilis ©, Pseudomonas aeruginose : 2D. Stephylogoccus aureus ce E_ Streptococcus pyogenes © 4g. A.S0-yearold nan complet of recent onset diplopia. On examination he 78 fable to 100k ; { Frmaes andl downwards with his Fight eye: Te reat likely diagnosis is a lesion of the: : a A. abducens nerve [ @. ciliary ganglion } 4 C. nasociliary nerve oculomotor nerve : » EE) tochlear nerve 0 i i j diy. AST-year-old heart trangplant recipient is keen. fo join the cardiac rehabilitation programme, i l Which of the following factors iS most likely to increase cardiac ‘output in this patient guring me ‘erate exercise? A. Decroasee. egative intrathoracic pressure .B. Decreased venous tone : Ci. Decreased ventricular compliance G) increased alrial filing * increased intrapericardial pressure 9 1 i | i { { ‘ 448. A 69-year-old ran with rectal cancer undergoes 1Ow enteriot resection. ‘The {ives is digoas fee. The pathology of his tumour WAS Voported as “2 farge tumour invading fromthe rectum into the mesorectum. TWO nodes; of 24 lymph nodes, close to the jumoyr were positivestor,. garcinoma on histology. The tumour was Complotely excised.” Wal is the ‘eorrect | pathological staging of his tumour? : A. Dukes' A B, > Dukes! B a = C. Dukes’ © : D. Dukes’ D Ee.) T2Nt 419, A2t-yearold man is admitted te the Emergency Department with 2 tab iary fo his right eee? pulse rate is 110 beatsiminute and Bead pressure (s 85/40 mmitg. Chest xray crews a large right haem. horax and a Vey on A right apical pneurnthorax. Which © the Frat substance secreted in the process paging to inereased renel reabsorpiion of sodium in response to the avove injury? Angiotensin | Angiotensin il Angiotensinogen “antidiuretic hormone Renin mooeP 120. 124. During the seconé (proliferative) phase of wound healing the predominant cells in the wound ctenase fibroblasts, This cell is of mesenchymal orian ‘and produces the matrix and collagen ae god to strengthen the scar. Gross linkage of collagen requires hydroxy-proline and hydroxy-lysine residues, which requires @ Specific vitamin to be available in sufficient quantities. Deficieney of which vitamin results in coliagen that is unstable? ‘ Vitamin 82 (riboflavine) Vitamin B6 (pyridoxine) Vitamin C (ascorbic acid) ‘Vitamin D (cholecatciferol) 8 Vitamin E (tocopherol) moose | A62-year-old woman presents with @ rm irregular mass in the upper outer quadrant of he right breast. This appears malignant on rn amnmography, and fine needle aspiration (ology, fe re,otted as C3. Which isthe frst lymph node fo which the tumour is most likely to ‘ metastasise? i A... Initial node, B. Primary riode Cc. Sevondaty node D. . Sentinel node 8 E. | Virchow's node Which of the following statements is rue with relation to cardiac muscle? Contraction does not involve retease of Ge" by the sarcoplasmic reticulum Contraction is triggered by an influx of Ga throagh the sarcolemma Excitation is passed from ono cardiac cel 10 another through desmesomes Hypertrophy is brought about by division ofexisting cardiaccells * ¢ There is capacity for regeneration a moony a TD-year-old rotred farm worker presents with 2 sealy lesion on the back of his teft wrist Biopsy shows enlarged pleomorphic squamors gells with mitoses that have breached the basement membrane. What is the name of this process? A. Dysplasia B. Hyperplasia C._ Hypertrophy a Melaclasia €) Neoplasia A 45:yearold man has established cthosts. ALa fofowNP appointment a palpable spleen. Four fngors’ breadths below the costal margin i ‘noticed. Full blood counts have shown @ persistent thrombocytopacnia ‘Bone marrow examination shows megakaryocytic Pyperpiasia, What is the most tkely cause of the thrombocy‘opaenia’? A. _ Ineffective production in the bone marrow B. _ Platolet destruction in the bone marrow Platelet destruction in the fiver Platelet destruction in the spleen) 0 Dlaialet storage in the splee® 2e for congenital nthe pos! splenectomy s carrying out an elective rabilise the spleen from 425. Asurgeon is il be divided in order to ™ structure wil A, Gastrosplenictigament B. Lesser omentum + (GC) Lienorenal ligament 1D. Phrenicocolic ligament E._ Short gustric vessels ic pain and a rig due to Excesem ted with epigastr ulcer. Uleeration é gastii 426. A 60-year-old man is acai with a perforated duodenal by increased levels of: A. adrenaline (epinephrine) calcitonin & 8. €) glucagon of 0. secretin. E. somatostatin yf her spine has st 1 with kerowen tumour of .e, The tumour Is snl and rectal ton \lowing vertebral levels is th A 66-year-old womer experiencing decreased at ere gullaris. At which one of the fo locates? ? 4127. ry of unilateral two-month histor in the testis wit ian presents with & neous mass witht sq heteroge! Jevated aloha fetorr 428. A 26-year-old mé ul sound sean show’ blood test show" an el spherocyto terior abdominal rotein eval. Which of the on en Oe wa wt .¢ tumour most likely testicular swe ith surroundin following is the mos sis. Which wall? ddomen. He is diagnosed nlactivily may be caused ran felling. AN ig fluid. His st like | - t diagnosis? | ‘A. Lymphoma : B. «. Orchitis c. Seminora | + (G) Teratoma ; : : Tuberculosis 8 to9. 60-year-old woman presents with = NAc of faecal incontinence over the past few yee ‘ She Pad a prolonged and difficult rst task, Ot spout 20 years previously: Physical ees hination reveals a relatively lax ancl ‘rincter. Which nerve is likely to Reve been 5 damaged in labour? i A, Autonorie nerves to the rectum B.. Genitofemoral nerve c. Lumbosacral trunk - é Obturator neve » (E} Pudendal nerve oe a 29 30. A 70-year-old man is admitted to hospital with @ 42-hour history of a painful white leg. A femoral artery embolus is shown on angiography. He Is an insulin-controlled diabetic and : takes ibuprofen for longstanding osteoarthritis. Twenty-four hours after emergency embolectomy his blood pressure is 90/60 mmHg and. he passes very dark brown urine. A dipstick shows myoglobin and traces of blood. His blood glucose is 15 mmol/L. (norma 4.0-6.0), urea 12 mmolll. (normal 3.2-7 5) and creatinine 180 pmol/L (normal 35-110). What is the most likely cause of his abnormat renal function? A. _ Drug induced nephropathy B. Hypovoiaemia C. Methaemoglobinuria 08 OSs Rhabdomyolysis ey E> Uncontrolled diabetes 131. An 86-year-old‘we an, with a history of partial gastrectomy, is in the high dependency wait os for a chest infetuion that is being treated with antibiotics. She complains of difficulty swatiowing. Endoscopy shows mutiiple ‘white plaques on the mucosal surface of the oesophagus. Biopsy is most likely to show: A. acid reflux oesophagitis B. Barrett's cescphagitis 2 :C.1 candida oesophagitis | D: eosinophilic oesophagitis E. herpes oesophagitis a 192. A 65-year-cld woman has been given a totat of 5 mg of morphine/over a 30-minute period for pain relief following a femirai neck fracture. THE*ollowing observations are foun ‘on 60% oxygen is 88%; blood pressure $0/60 sniii { fg; respiratory rate 6 breaths/min Immediate intervention should be SpO2 The A. high flow oxygen B, intravenous fluid replacement f « (GY intravenous Natoxene B. monitor her ECG 08 E. tracheal intubation ; 133, The following drugs are used lo treat patients with peripheral vasculer disease. They all reduce the rate of myocardial infarction and stroke except: A ACE inhibitors 8, antihypertensive medication ; ©. aspirin D. cholesterol towering drugs Qo : +E oxypentifyitine : i 134, A d-year-oid bo, is brotight to the Emergency Department with a painful right elbow. His Mg father states that he was swinging his son by his arms when tha pain came on suddenly. Bf An X-ray shows that the radial head is displaced from its usuabtosilion. What is the ligament (of the proximal radio-utnar joint) that holds the radial head in place? * Al Annular figament B, Conoid ligament C. Suadrete figament D, Radis ligament oO £. Ulnar ligament 135. A 35-year-old man presents with a three-week history of low back pain and a three-day history of pain and weakness In his left fag. Physical examination demonstrates numbness | ‘over the posterior aspect of the left calf exiending {0 the lateral aspect of the foot. Loft ankle reilex is absent. A magnetic resonance scan shows compression of the leit S1 nerve root. ‘What is the most likely responsibie structure? ¢ A. Annulus fibrosus | ay Anterior longitudine' igament : » (Ga fetis. pulposus a : E, 3 * longitudinal igamerit i Vertebral end plate i : i END OF PAPER f E newt secondary he following is ty hypovolacrnia in Cais p respot ated venoconstriction ©, Decrease Ds | strial netriny enal Suid retention: in-ungietensin system acti cot EJ 4, Histology ef a discrete palpable lump in the broas of & old woman bas ol apgetins metuplasia, epithelisl overgrowth and papiliary proyections. What i ological prousss? howe, ythoids ke Pimwcn oF A BS-yeur wo: peeselits with recurr ic ulceration, $ t pept ‘he ig 92 proton pump inhibitors sis previously received Helicobacter pyleré eredicmion therapy three . snouthy age. Which: ‘oF she following is fikely to be raised on venous bloud testing? A Che wb. Gasrin yystokinis 4 tid vault hoy is aduitted fhe assstcnant af ental blond! loss. His no describes iis as bright red blood ie the toitgt pan. He has no pain em defecatics no tunily i i i A. Adeuommous pelyr Be bavenile fac most * peosents with a painful woxtel tras been feeling rater tre lateval part of bis neck 2s ter veut. ane coe camnected, [he mst likely cause yuerohd ure his ony comploi cxainination be has a iange rubs gaailer eviases along his joi spasm and WE sbbeey. Esy. ° Seles Stew the spusoular » tojfivelie is due vo pressure ansa ceric: | cervical plecus branch Qo | craiat accessory Bet spinal 4 i seynauold woman is diagnosed wit besashomcer i she had hor, first child awed 40 years ‘During lactation, she eloped u beeast abecess tant necessitated surgical intervention. She bas no family z which af Ue Futlewing is unifteant risk factor for pregneney EY position, 88 Jarchie end anenopaase | se yresst OBS ical slate clinic with weight loss of 3 kg over tilations..On islaeni to the nutpatie | d woinan is 7 st Sasyomibs: She s complains of aasiety yang tines ane piptallehs x on, there is « swoiling n the sl whoich mpies on swaliowin [ Shieh afte faltowing is the most likel athology uederlyng tbis presentation? wh. Graves’ disease A eshimnoio's thyroiditi % : sig tw the thy carcinoma : §, A 69-year-old man bes been admit dency unit following 20 eneral Biaesthesia, He was given 2 mg of iatralecai pane anterior resection under 8 2. Oa exantination hi Yorks pole and drowsy. Afterial blood gre esa! aes — cc A 32-year-old women lias a pigmented jesion excised thom her left calf The hislopathofogiced diagnosis i: melanoma in situ, h is completely excised wath a . {om miaugin. What isthe next most appropriate management? i s Felucation about skin self-examination and discharge from foltow ap | _B. Elective inguinal lymph node dissection 1 1D. Retnoval of any other pigmented lesions u FB. Sentinel lyonph node biopsy « 28 io. 4 60-year-old‘man with a past history of angina undergoes an uncomplicated operation for an inguinal hernia. Postoperatively he is found to be hypotensive, tachycarlic, aid has a raised jugular venous pressure. Wht is the most likely underlying cause of his hypotension? , 5 7 A. Ruduced afterload ‘ B, Reduced parasympathetic tone : C. Reduced preload 08 1 D. Reduced stroke volume | E. Reduced sympathetic tone {4 iI, A 4.year-old boy presents to the Eme!gency Deparlinent with a pvo-day history of sae fever, difficulty walking and is unable to. weight bear on the sight leg. He has been on oral Anioxyeiltin 250 mg three times a day for a chest infegtion aver the ast five {i days. He is a temperature of 394°C. He does not allow examination : aud keeps his right hip flexed and adducted. Blood tests are: white blood cel! count (WBC) 18.3 * 10%dL, C-Reactive protein (CRP) 146, liwemoglobin 11.3 g/dL. Whar jg the mast likely diagnosis? Y A. Acute avascular nestiosis of hip 7 B, Dislocation of hip C. .Perthes disease D, Septic arthritis | E. Slipped upper fensoral epiphysis Hl 12. A 34-year-old man is admitted to:the Emergency Department ‘vith a head injury. Ox 4 examination his Glasgow coma scale is 9. A CT scan of the brain demoustrates an extra-dural haemorchage. Which of the following arteries is the most likely souree of | the bleodidg? L A. Anterior cerebral \ B.-Middle cerebral! Ee. wc, Middle meningeal { Th Posterior cerebrat B. Superficial temporal A G5-year-old man has s history of transient ischacrsia attack soiid endayierectomy. Whick one of the follow He is due to undergo is Ute of the interna! carotid . A, Begins at the levelof the sixth‘cervical vertebsa $/" * \ are 5. Divides into the anterior, middie and posterior cerebral arteries PSY" Co et C. Gives off the ophthalmic artery __ WD. Is accompanied within the’skull by preginglionic sympathetic nerves eae TE, Passes through thelToramen ovaleCave Fe tract bh ‘ : i 14, A 25-year-old male athlete is in training. At rest, how many litres of blood per minete does his beart pump our? i A. 09 +B. 2103 ; : ¥C. 5106 | YD. 810 10 4B. 15 t0.20 x 15. A 26-year-old man presents to the Emergeaty Department aiter sustaining a glass | : cus to his xm, On examnination there is a 10 om Jorigitudinal laceration on the enterior aspect of his upper anm. He has symptoms suggestive of ulnar nerve injury. n On exploring the upp&r part Y'the arm we would expect the ulnar nerve to be: A; antetior then medial to the brachial artery is 2 medial ther: anterior to the brachial artery I %S YC. medial to the brachial a-fery D. posterior thea mediate the brachial artery 9 : Il E. posterior to the brachial artery | a gafeese oak rmaeiled fy | 16. A 55-year-old man presents with acute back pain following a severe road traffic accident, Nevrological examination reveals lack of sensation of the umbilicus and below. What is the spinal ievei of the neurological deficit likely to be? Te Ty Tio TH v2 Heo Sui > | : 4 { seater tienes RACES 2 iicesmnrcetin 6. D. impaired glucose tolerance year-old man complains of being thirs' ‘go to the toilet, His weight is 95.5 k following results: sting plasma a1 bess OO uno. hhour plasma glucose | (iP mmovie “FBS 19 A. diabetes insipidus : Two howd B, diabetes mellitus C. impaired fasting glycaemia B. normoglycacinia 18, Jna9-year-old child with cellulitis of the hand, wh! muogiobulin preduction is correct? i A. IpD precedes {gt production B, JaG precedes IgA production C. 3gG precedes Ig production D. IgM precedes JgA production s, [gM precedes [gG production in 19. A 60-year-old tian presents to the Emergency Department with spistanis, (Vig Diecding is identified as Little's area aac re vessel is navst likely reeponsible for the bleeding’ height #94 inmllg. An oral glucose ioleraiice test was peitérmed and produced the Drabtrs + LEGS? wg up in the middle of the 1.65 m and blood pressure, ynosecd if * > mnt[e and [Ir ~~ bn 1S abaye [-L mmcth @ fovrancle © mood | Foon i betwen PTE ich chronological sequence of 8 7 he source Ives with direét canter . Anterior ethmoidal aytery |. Infraorbital artery | = superar Labret ia Middle sneningeal arfsry ands Eghonuraleak Splienopalatine artery 0. Gveoim. potuhne Supratrochlear artery" Fphorep ub Te obey of ‘eprhans ” phevepotakere 1 dPhetie polabrre - hageal reflux. Endoscopy confirms the 20, A S-year-old sum presents with oes presopce of @ hiatus hernia. The desophagiis passe verieinal fe 5 through the diaphragms at whieh co ps Vat 'O fo America - TiO 3 wf Dp. TH nw ; ET2 | peer reer di swounay with emphiysern: blood gas spsults, “6! it Spray mre f VB. Partially compensated metabolic acidos) L “C. Payiialiy compensated réspiratory acittosis 1D. Uncompensated metabolic avidesis + Uncompensaled respiratory acidosis. F radiowurgery on a cerebral srteriovenozs malformation. Four pins sec tightly through the scalp to the outer table of the skull, wo anteriorly, two posteriorly. On insertion of one of the posterior pins, arterial haemorrhage is 22, A 2¢-year-old man is having a stereo! fame filted to bis sieull pr | ‘od, Which artery is most likely fo have boen punctured? es ‘A, Ths ascending pharyngeal artery cin PONE 7 een fe ridlbn 3. the middle meningeal bow Tuacl » : B. The middle meningeal artery fw Woes empont C. The occipital artery a LB He povenoronebeat atery whines Segre avafed. he pdsterior comununiceting artery vear-old man prosents with a two-month history of unilateral testicular ng. Ap ultrasound scan shows 2 heterogentous taass witha the te: i i surrouading Sluid. His blood test shows an clevaied atpha fetoprotein level. Wh the Collowing is the most likely diagnosis? 23. A2 sich of Seminome 7D. Teratoma tuberculosis old baby presents with increasing respiratory distress. He was born et Syidlnal delivery. On examination he bas cyanosis of the lower Enibs one saatetieshifatory brdraiving of the chest. His femoral pulses are absent bilaters | fed he has been anurie for the lavt,12 hows, Pulse rate'is 140 beats! re t and liis blood pressure fs 60/30 mabig in both upper limbs. What is the me diagnosis? : ; b A. Hypoplestic leftizeatt syndycine ©. Antesrupted aortib arch C. Pulmonary atrésta and ventricular septal acrect 1. Transposition of the great arteries auesia ond woman presents with @ history of right upper guadtan! pain 39 She ropouts that ber rine was dark in colour ans that’her stools are alt to flush. ‘Which of the following explains the dark urine? w. Increase in conjugated bilivubinuria B. Increase in unconjugated bitirubineria C. increase in uxea excrétio ____D_Inctease in urinary ufobilinogen 7 EY Reduced enlecohepatle ie Salt vircatation 26, Aut 80-year-old woman, Who hes suffered a fall, is found lying o® the floor, where shows that she has a core she has.been for over 12howis. Initial assessment emperalure of 28°C. What would the expected eatly physiological response to her body temperature be? A.- Inereased fat inetabotismn ” : Incréased constiiction of peripheral blood vessels Increased hypothalamic set-point : . Increased metabolic rate Increased thyroid activity 21, A V2-year-old boy presets to the Emergency Depaitment two houss after helping the father out the grass. He complains of rhinorthoea, tehy eyes, sieezing and « trched nose. He fs apyrexial with a haemoglobin of 12.7 g/dl. and white blood cell count of 6.8 * 10°/L with a raised eosinophil count, Chest X-rsy is'clear. Which immunoglobulin is most likely to cause this reaction? A. IgA B. JeD ¥C. IgE . IgG rE. IgM + leg. pan dg te weprokapsed untae di by coughing and syeczing, radiates F the dorsiflexors of 28, A 45 Jearold mam presenis will backache and} intervertebral dise seb i aggravate (o.the dorsum of the foot. O2 «| the foot. Which herve root is most likely to be inv joived? ATR + B13 : : ck | YD. SES Q- E. $2 ¥ #4 75-year-old woman, who has @ carcinoma of the upper rectum, undergdes an : anterior resection, The arterial blood supply of the upper rectum: arises from wisich nm of the following? A. Coeliac entery : B, Meocolic artery o vC. Inferior mesentesic art E. Superior mesenteric artery ! : ee A Gf-year-old man is undergoing an abdominal sortogram. 4 stenosis is ciemonsirated in lateral aortic branch, arising at tue level of the body of the second iusibar vertebra. The stenosed vessel is most likely to be the A. coeliac artery B, inferior mesenteric artery , ieftyenal artery ~ a 7 D. second left lurmbar artery E.. superior mesenterie artery 31. Add-ye id man is admitted to the surgical day case ynit for repair of his left { inguinal bemia, On examination he is noted to have ditfuse skin taniaing, spouty igmentation of the elbows, aipples and buttocks, aud pigmentation of the soar from ‘ a previous right inguinal hernia repair, Three hours afier the operation he becornes { severely hypotensive. What is the miost likely cause? 5 A. ACTH deficiency ald gi “" : i vB. Adrenal insufficiency — © - . C. Growth: v 7 f D. Potassium defic ol eu £ \ #. Thyroxine de ; | 32. A 19-year-old woman presents to the Emergency Department profoundly hypovolaemic lsaving fallen from a horse. A postero-toterior (PA) chest radiogzaph { shows a fracture to the medial third of the left clavicle. Which of the following l vessels was.most likely damaged? ' | A. Brachivvephatic tank | B. Lett axillary artery a C. Left common carotid artery \ #0. Left subclavian artery Left vertel HI Pee 7 Pato A. External dblique apdueurosis ¢_ exk- sperma. Forse} pe fein Interna! oblique muscle ig ti eee tee Reotus abdorinis muscle Rectus sheath oo} 34. A 7O-year-okd woman is in th r Blood ges shows | Reduced seasitivity of hich receptors is most likely tv he responsible for this blood [ picture? t A, Adrenergic receptors f B. Baro receptors C. Centtal cheme-receptors fl 7 D. Jreceptors { E. Lung’stretch-receptors is , L 5 -yeas-old man presents with pain in bis 18f serotun. A diagnosis of varicouoel [ is made, Which vessel ig involved? A. Femoral vein { B. Inferior epigastric vein ; c. D. | { 36. A 60-year-old man, who is a heavy smoker, presents with a 10-day history of frank t painless haeynaturia, Hlis prostate is slightly enlarged on rectal examination. His haemoglobin 's 11.3 g/dL, creatinine 84 pmol/L and prostate specific antigen (PSA) f is 3,1 np/al.. What is the most likely pathological process? 1h, Bepign prostatic kyperplisia : \ 1B, Bfostate cancer t 4 “VC. Renal cell carcinomd / “es DT ausitional cel! cardinoma of bladder | “Urinary tract infection 3. A 34-yeur-vld woinan clinteaily, radiologic cancer at ie age o Which g: resents with anit Y and histologic: £58 and her grandmoth is most likely to thevight breast whi st. Rer mother died of breast ied of ovarian cancer at the age o: is be involved in the development ofthis woman's: "ATERCAT A "pia inn presents vith a noa-tender swelling in the tight hemi-seroturn. At { ‘eration, the hydrocele vac is opened and 400 sat of fluid is drained. Which Anatom: Darwos mi Patent proc ©. Testicular capsule shica albugingy vS. Tunica vaginalis 39, Yert-old mau is brought ty the Hie eecklens, He has cbvioks airway comp: { which reqaines a surgical erway. The su { 4 tracheostomy incision ere mid ch of the following? ' : L cond” ant Fi ‘opathalogy report indicates average five-year su'vival raz cnt ‘ AL 169% + 1 B. 204 . 45% D. 70% * Lo BB a ‘ #2 stcture swrounds lie fh iS vaginalis 108 gency Departament following «j ise dus lo menthifacisl hacmonthege Ace lendiiukee used to focatise the opti Sehveen ths suprasternal noteh and? sd traifie Hof seseetion fore high rectal caroinome. Te the lesion is Dukes? stage B. Whe is the appro: hese lesionis? | t ; k a eraser ney eee sto the Emergency Department after acid ag his hand on a glass bottle. Ou exemination there sypotbenar eminence with loss of flexion in the distal incerghlangeni joint of the lite finger. What is the most likely tendon te be injured? ly u a2em 7 and inces > lncafation on the in A. Flexor digiti minim brevis LB, Flexor digitonim profundus six nivaths ago. A left intemal mammary artery graft was used. He now complains f ] ofengina on pegging his laundry on the washing line to dry. Which of the following lesions best explains his symptoms? A. Micto-emboli from the le comraon Micso-emboli from fhe le°t internal carotid artery : Stenosis of the subclavian artery at the level of the costo-cervical taunt. Stengsis of the subclavian artery distal to the insertion of scalenus anterior Stevosis of the subclavian aitery proximal to the first branch . a f] ° : 4% wclic man presents to the Emergency Department aud 's found to foroxiend the wrist-thumb and fingers of bis right Hand. te te have weak éxtension of the right elbow joint and loss of sensation: ox dhe dor ¢ Csi weig space. Which -rerve is most likely io bnve been injured? A. Median nevve wt” J were Musculocutaneous nerve Posterior interosgeous nerve Radial nerve 7 ~ a ‘Ulgar nerve S.year-old man pre Neurological examination reveals lack of darsi-flexion of the ch of the following spinal cord segments are siost likely lo be ix ts with acute hack pain following a severe road Uafiic G~-Plenor- ditions cuperficielic + i D. Fourth palmar interossecus : B. Luubuical . ] 42. A 75-year-old nea, whe sthokes heavily, underwent coronary artery bypass. geafling L 45, A 2i-year-old man js admitted to the Euiergenicy Department with a stab / his right chest. Pulse vate is 11 beats/minute and bivod pressure is 8374 Chest Xap shows a large right haemothorax and a very small right apical pneumothorax, Which is the first substiiice secreted in the proicess leading to increased renal reebsorption of sodium ip response to the above injury? A. Angiotensin I | B, Angiotensin 11 “ .C. Angiotensinogen | 78d. Antidiuretic hormone : pXB. Renin : f ; 46, A Zl-year-old man bas beea stabbed in the back of the knee, dividing the popliteal artery, and is undergoing repair Via a posterior approach. Which of the following ed first when peste deep from the skin |, structures is most likely to be encow: { : incision? {yaw SSP Ap A, Popliteal artery + | Paks 7 7 , B. Popliteus muscle - pep: ENS : l C. Popliteal vein = peopl. burco D, Tibial nerve E. Soleus muscle es i nett a [ = arabe ey = Conemne Palen | 47. AT8yeae-old woman presents with Inidary urgency and incontinence. The external *” . urethral sphincter is innervated by which of the following nerve’ roats? sep lores | AO L3,4&5 . Ea B.L4,5&S1 5 C.L5,S1&2 S1, 2&3 YE. 82584 | 48, A 40-year-old wornan presents with a parotid muour. A biopsy reveals perinental - invasion. Which is the most likely pat hology? . | JA. Acinic cell caivinome L 7B. Adenocarcinoma ie H C. Adenoid eystic édeinoma zl D. Lymphoma t “> WB. Pleoinszphie sativary adenoma x | 49. A 19-year-old inan was agsdulted and sustained injuries to the tight side of his head 4 After iwo weeks he notices that his right eye is dry and it could not produce tears, From which ganglion da post-synaptic Hives mnie & Geviculate ganglion B. Inferior ganglion of the vagus nerve 5 WC. Olic ganglion ; eae { viper? Prerygepalatine.gadelion, oo. ¥ es upcrior eervied! ganglion wragal Mace Sa ais C Nie Plarne glands : mY C665 OL Nucy great eusinte { : ' YO ross dtevy “hate . Boast wna # 5 ju eadergoes a right iuguinal bernia reper under goveral anaesthetic ‘day patient, Fe has a nérve block afer the procedure Oi, recovery he has acaess of the sight feg. Which nerve has been affected? . Femoral nerve Gcnitofemoral uerve’ oO Uioinguinal nerve | . Lateral cutaneous netve cintionen i: 7 $1, Amen suffers a brachial plexus inju: Jn association with upper limb paralys : syndrome suggest involvement of? : { A. C2. & C3 roots B..C4 & C5 roots» o f . C6 & C7 roots t CBT roots}. E, T2& 73 roots On examination, "he hias a Homer's syndrome . Which nerve rovis does the Horn t 4. 60-year-old woman with breast carcinoma complains of difficulty chewing her food and is found to have numbness of the lower lip on one side, CT'scanning shows 8 suns metastatic lesion affecting the bony sleull base on:the came site the ip | numbness. Which foramen is this most likely (o involve? j A. Foramen caecum < rH . B, Foramen magnum : : v&. Foramen ovale ( meidilodon) wD. Forme rotundum — (reote ton] { E. Foramen spinosum ‘ : Which one of the following muscles is an extensor of the hip? ue fea A. Adduetor longus B. Gracilis ; . C. liopsoas D, Pevtigeus cE. Semitendinosus \ t 54, In order to expose the right axillary artery, a transverse skin incision is typically made below the clavicle from a point just laterat (othe sternal end ofthe clave to \ the dellopectosal groove, Which of the following structures would be sncountered ga { the dissection down to the vessel? 7 \ A. Laderal thuravic auiery B. Phrenic nezve ©. Suprascapular artery 0 D. Thoracic duet E. Thoraco-acror L 15 56, A SS-yoav-old extn presents (othe Emergency Department after collapsing, On A esusation is putse fs 124 beatsfininute, blood pressure is 60/50 mig, respinatory rate is 34 breathslinute and bis peripheries are warn. Wha js (he most Likely diagnosis. for this patient? A, Cardiac failure 'B. Hacmorthege aint a D. Pulmonary esabolus B. Sepsis . Ve 46, the posterolateral approach to a postéior malleolas fracture, an incision rman the caloaseal (Achilles) tendon and the distal Hbula. Whigh of the folowing strvotures Js at risk? : A. Deep peroneal (fibutar) nerye B. Saphenous nerve C. Superficial peroneei (fibular) nerve EF) B. Swalnerve *. co E. Tibial nerve 57. A-25-year-old wonnan undergoes an elective right thoracosropic procedure for A heeroat of tight palmar hyperhidrosis. Diathermy ig applied (o @ neural s iying aiterior to the neck ofthe right first ib. What is the wrest likely coxaplicetion to oscur as a result of this procedure? 7 t > Comperalg Hap eyehoree : A. Bradycardis B. Floarseness > prewne Ike syndrome = plowel Ha gors c, Homer's a 1D. Raised right hemidiaphragm E. Reduced right biceps tendon reflex a shopaedic outpatient clinic with a sixsmonth history of low, back pain radiating <0 the [atera] aspect of the left upper thigh. em verison and defecation ao normal and thote is no bitory of previons injary.,On are nation, left atera flexion ofthe spine is limited but a full range of bip Saar ents are observed, allow it 8 painful, Sensation is cltered oY! the fiont of the ines, Left knee reflex is reduced. Which spinal nerve is most tikely to have been 582 A 33-year-old mian presents to the oF compressed? AJ LI B, L2 C3 6 : LD. 14 7 ELS t ident, having and arupiured spleen, Three days {0 be cunfused, bypoxoemic and ciffiouit to ventilate. Hig ehial at 20/80 mmllg, regular pulse of 86 "729 shows bilateral diffuse lung ink dizgnosis? noteii vations shave a blood beats/minute apd he is apprexial, A chest x. sae jofiltrates. What is the most Ii ely underlyi XA, Adult tespivatory distress syndeamne farnsy I. Aielectasis ©. Bronchopneumonia ae a D. Pulmonaty oedema Pulmonary throrbo-embolisny 60. Onan ultrazound scan of Ihe [2° popliteal fossa when investi igatingsa swelling, which of i 5 ‘he following structures if closest to the Capsule.of the knee joint? f | psu is i L 7 La * A. Great Gong) saphenous vein VR. Popliteal artery ©. Popliteal vein D. Small (slion) saphenolis vein E, Tibial nerve 61 fon, he has a full range test aad a small effiision. Wh (0 give way. On 4 positive auterior draw Structure dimaged? of knee tinction. There is ‘at is the most lilcejy : W& Anterior cruciate ligament | . Lateral collateral ligamen’ ©. Medial collateral ligament D. Oblique popliteal ligament { Posterior cruciate Jigament Old tan required a partial! Bastrectomy Sor a large benign ulcer his. trie antrun, When be is reviewed in clinic six months later he compiains of . Palpitations, weakness and Sweating along with a cramp-like abdominal pain which“ | ra Atthin a8 hour of cating a meg, Usually fe has to fi ; c down for 30-40 minuics L Until the symptotis subside What isthe inost Jikely cause of his symploi pattern? ~ f A. Chronic gastiypucas <8. Dumping syadrome a. C. Delayed gastric empty D. Refiux gastritis & E Recurrent i leerati pa bet oid tnah presents with acute epig: 29 guarding Upper sbdowisa His tess» stilts are: Gamma GF] Bice 7 ikely esticiogy of this i Mleehslee. hepa AG, 4 bo ithissio—y Mob i a - ap is cetidaemis com 2. Bypocalcaemia 5. Mumps with ead stoge renal failure requir iptomatic: pail istone distase. Hier haemog na) defi vB. Epyrluopoietin deficiency C. Folate deficiency . Iron deficiency Vitamin B12 deficieney A 60-year-old man underore cy ureters are ident! Dladder watt? A. Anterior surfice B. Apey 5 fon it bia, dder carcinoma, h region of the blader do the ute HC. Loteral surfaces < Pala. and vomiting. ¢ he systemia hich ofthe. saeruaetie Nor met: Oo es an elective Jobin During surgery, ters pierce the a colonic resection for carci GF A 65-year-old ma joma 12 bours ago. He is iidwe passitig concentrated arine“at a rate of 0.5 ml/kgzht. Which eri sponse is : ‘most likeiy to have caused this? A. Deoreaied aldosterone release A. Decsensed insulin release i C. Decveased thyvoxine re ease ; D. Increased adreno-corticotraphic hormone (AC 2 ¢ © lnicaeased vasopressin release” A) 4) fl VU ¢ g DM). [ G8. A 28-yeas-old man presents with anischign stal abscess. Where is the abs cavity likely to be sites? A. Above the levator abi B, Between extemal and intemal ana sphincter . ©, Lateral to the ebtarator intemus —, D. Medial to the internal anal sphineter WE. Medial % the pudendal canal . oO 69. A 45-year-old nan falis on his oulstretched right Kavi. Examination vevesls § tenulgrness in the anétoinical sauff box, Which one of the follow'ng tendous fo I eourldary of the anatomical snuff box? , . Abductor pollicis brevis. arpi radiaiis brevis Hxtensor caipi radialis longs 3. Extensor indicis W [ Extensor pollicis longus. (medi 4) t + 79. A. S5-year-o}4 wontuet compisine of cramps and Gaptin hours following 2 sub total hyroidectomy. Her vital signs are stable. Investigations reveal a normal white-cell count and aosmel hacmoglobin, The Na’ is 132 mniel/L: 43.2 mmol/L and Ca” 1.60 mmol/L. Serumn albumin is notmal; The next sppropeiate step is intraYvenous infusion of: A calumeblodte <7 TS\7 culeacvr Lelcrne j i magnesium sulphate ; potassium chloride sodium carbonate . chloride : BooP 1), 7. A 40-year-old women prese: internal and external anal se damaged? First sacral nerve-root Obturator nerve, Pelvic splanchnic. nerves 7 Perineal nerve * 5 Padendal nerve vow \ 72. Abdominal free fluid will collect operation with first? in the lowest part of the peritoneal eavit ‘he patient supine, in which of the followiug yl! tne fen AL collect eA: Hepatorenal pouch Caf ynovsSon) B. Left subphrenic apace ©. Lesser sac D. Right parscoliclgutter i Right subpirenjc space 13. A 45-year-old homeless man presents with a cough and weighit loss over three months. On examination his BMT ody Mass Index) is 19 anid he has reduced, breath sounds in the right upper zone, is chest ray shows a cavitating lesion in the right upper Jobe. He undergoes a bronchoscopy and a brovichial biopsy, The biopsy shows featureless necrosis surrounde: 4l by epithelioid niaerophages and piant cells. Which of the following is the most likely diaenosice A, Actinomycosis B. Bronchiectas C. Sarcoidosis D, Squamous cell carcinome E. Tuberculosis Seabees aie RacOld mia is being assessed with a view to being alive related Kidney Agage Whis! UgAUGHIS IS mest accurate for fottu ‘measuring the glomerular filtration tale? A. Creatinine clearance B. Glucose clearance _- = ©. Inulin clearance 7 D. Pay Hef mother and maternal 2s, Whichof the followin, 4 M-yearsld pregnant weinan deveiops & swollen a hac! a similar problem during their preguanc tests is likely to be positive? A. Antiendomysial antibo« BB, Antimitochondrial antibodies C. Antinuclear antibodi D. Antiphospholipid antibadies 2 4" y! Antithyrogiobulin antibadies pros WAL 16. ‘A Teyear-old alcoholic man is known to suffer from chronic obsteuctive pulmonary disease (COPD). He has now been diagnosed with carcinoma oP the urinary bladder fandihas a bislory off gastzi¢ leer. Which of the following agents is most likely to be responsible for ali these problems? Alcohol Cigarette sm Drags D. raphic ‘A Seyear-old boy is admitted to hospital with severe vomiting, Radic creas. Which of examination and history zeveals that he is swffesing from anata pa structures is constricted? 7 thes A. First part of duodenurn #B. Second part of duodpuum C. Third part of duodertum 1D. Proximal jejusum E, Pylorus of stomach + ‘AS, year-old sian presents with a 3a nomiender. A cough impulse is elicited. At operation, an indireé? inguinal hemi is I ring is a defect jy of ite following abdominal cling in theight groin which is found. The external inguin wali layers? C. Internal oblique muscle B, Transversus abdominis, 2. Transversalis fascia aa Yeat-old woman suffers from spina bitila and is confined to awh legs ave not the legs? cele’ lly developed. What is the pathological process whieh hee cents A. Apoptosis ZB. Atrophy C. Hyperplasia Oo : D. Hypertrophy WE. Hypoplasia ~ : Sat-oicl manjpresents to the Emergency Department wi Pains. A postero-adterior (PA) chest radiograph is performed and ig normal. Immediately infesig: (caida to the outline af'the sortiy fanuckle was a further se with a convex border, What is this structure? h nea-specifi A. The left atrium - RB. The pulmonary tunk 6 €. The oesophagus x D. The right atrium BR, The superior vena cava x = 2car-oid wonian presents tothe hospital with jrundice, Sé uncdes goes paleo sca0.and is diagnosed with a hepatoccililar carcino: clea ing fobe of ber Liver. Which virus is the most Jikely to be implicated? Ie fast AL stein’ Bare vilus (EBV), vB. Hepatitis B virus BY) 7 C. Humad immunodeficieney yi D. Human papilloma vicus (HPV) 8, Huon T ceil (ymphotrophi virus LYE + An 82-year-old mau has complete occlusion of his ipfertor mesenteric artery apuieBraphy but no symptoms or:signs of colonic icchaemia, Which agar follow inicio {he most likely addtional source of blood supply to the te itary of the inferior meseat, Pee B. Left gastracpiptoic > 8 Middle colic old man with documented Type } diaketes inellites presents to the ‘al ouljsaticat clinie witi a three. ‘story of right leg pai brought on by tres. He has no history of back pain, He has heen @ smoker for 1¢: years (10 cigarettes a day); He slates his alcobul consumption to be 30 units per ck. On clinical examination significant findings include weak right foot pulses Wt altered sensation on dorsum of the right foot. What is the mos likely diagnosis t from the following list? Autonomic neuropathy a et : Femero-popliteal stenosis : C. Neuropathic joint disen j D. Polyneuropathy l E, Radiculopathy \ 84. A 35-year-old man presets with a three-week history of low back pain dnd a three. day history of pain and weakness in his left leg: Physical examination demonstrates numbness over the posterior aspect of the leit calf extending to the Jateral aspect of” the foot. Left ankle zeflex is absent. A magnetic resonance scan shows compression of the left S1 nerve root. What is the most likely structure compressing the nerve? A. Aumulus fibrosi B. Anterior iongitudinal ligament C6. pulposus, f <0. Posterior jongitidinel ligament E. Vertebral end plate at 85. A 72-year-old man who js a sinoker presents to bis doctor with a second episode of | Henk haematusia. When fn ultrasound scar of his urinary tract was pat formed it showed a solid lesion it fie right kidney, What is the most Jikely pathology underlying this preseistatlon? A. Angiomyolipoma v4. Renal celi carcinoma ; © Renal cyst oO D. Renal oncocytoma E, Trangitional cell carcinoma Hoan tle dat histen-efteauena Wo 86, A 20-year-old man presents with abdominal pain and shock. He is found to have by the most likely predisposing cause of his splenic rapture? cA. Epstein Serr virus infection B. Human immunodeficiency virus (HIV C. Measles virus infection D. Mumps virus infection E, Varicella zoster vi infection“ infect x diarrhoe: at~éld vioman which 8 grenilometous clinic with a three v rectal biopsy, the hist the most likely diagnosis? month history of tology has shown’ , i A. Améobiasic [ +B. Ctobn’s disease ©. Sercoidests 2 Z 5. Tuberculosis [ B. Uloerative colitis A S6-yeat-old motorcyclist presents to the Emergency De; involved in a road uaific accident, He is ¢ smoker He is conscious (Glasgaw coma score of 13/15), | sod bitaihing. Hi is found to have an ones. partment after being | ‘td knowin (0 have Ayperiension, ; aud maintaining his own sirway 4 ight femoral fracture With nonnab distale vz pethtal catheterisation drained 250 ml ih he drained oly 10 mt, Which of the A. Blocked catheter ; B. Cardiogenic shock | ©. Hypovolacmic shook, 9 NOD, Ruptured bladder E. Septic she 89. A 35-year-old man is edenitted init Pancreatitis, y \ wieice! igh dependency ait. After 48 hou | “th elevated recpiratary rate and ners tespiratory Saihue, Which of the follow regulation of respiration? which leads t admis develops brett sing hypoxiu. You suspess Og Mechanisms is resp Sion to the A Brainstem receptors produ CO: concentration in medulla =. Hlevsted bicarbonate levels in ces 7 3. Aydiogen ion diffu E. Aypoxi. i Voluntary breathing spinal fluiet % sion across blood brain farrioy © stimulation of chemoreceptors 90. Y0-year-old mal undergoes tronsuretbral reseetio fe tumor ics ver the lateral side wall of tee sae iltee. Use of the diathermy sudden: hear of stimuiated? CAUSES the pte 'P 80uCW muscies, Yat ve has been A. Femoral nerve Genitofemorat & Sateral semoral cutaneous jerye TR. Gracilis tear 9 SA lavearold ei ‘he Following statemenis'is true’ with relation fo cardiaé muscle? ‘Contraction does not involve-release of Ga2* by the sarcoplasmie reticulum: Contraction is wiggered by an influx of Ca* through the sarcolemma 3 Excitation is passed from ose cardiac cell to another through esmosoines Hypertrophy is brought about by division of existing o diac ceils 5. There is capacity for regeneration 92. & 38-year-old man presents with searing left-sided loin to groin pia. Taves seveale a radiodense opacity, confirmed to be tigetion . What isthe most likely compo: in the ureter on intravenous urograpa sition of this man’s uretes AL Ainmoniury magnesium phosphate _— #. Calcium oxalate : C. Cholesterol D. Cysteine d . E, Urate 95. A 25-year-old man is pldying football wha he complains of whilst sprinting with the ball. On examination he has tence posterior thigh. He is unable to filly extend the knee due Hikely injury? postetior thigh pain & | mess initize Lower lateral to pain, Whet is the most wt\. Biceps femoris tear 7 i ©. Gasteoenomius branosus tear Semimeni Semiteodinosu: Seyear-old ina is ase wound ie cleaned and a s with 4 pulsatile swelling ulted and sugtai ioople dressing app Af the site of the inj stab injury to the lek jicoin. The i red. Six months later the patieni returus ily. ‘The most probable jusian presen smn of theifemorat lymphovele myeotic neu saphena vatix hosed ntercostal space. Pathology A. Aortic valve > B. Duets arteriosus yvC. Mitral valve Putmronary vatve t ive ‘Iriouspid y of witich structure is the most ikely canine? j | id mon is admitted having been Stabbed in the.anterior chest, On [xemination he is alert and coherent, He has Congested neck veins. His pulse ig 149. + beals/oninute and his blood preasure 90/60. inmbly. He has rosial breath sounds What is most likely cause of his cardiac arrest 15 minutes later) LTA. Cardiac temponaide “—B. Congestive cardiac f ire ©. Hypovelaemih , . : D. Tension pevmothorax ®. Ventricular arrhythmia 97. A 20-year-old woman presents cee Rlth sbdorninal pain. Irvaging reveals aright Ovarian cystic lesion which is excised Histological examination shows a cyst lined by keratinising stratified squamous (bithelium, Areas of ft, muscle, thyroid ane Feural tissae,are seen in the wall. What is the appropriate pathologi forthis. lesion? > desigeation « A. Cystadenoma B. Dysgerminoma, C. Meseachymoma D. Squamous cel) carcinoma v7 E. Teratoma , | 98. A 2l-year-old map comes into the Emergency Department with a severe he. and neck stiffuess of recent onset, You suspect acute thoningitis and decide immediately to treat with antibiotics and perfor a lumbar puncture, After your thied sain {2 obtain berebrospinal fluid (CSE) you notice tran thé fluid obtained is tengd ted. However during the coflection of CSP ine ee containers itis noted "hat the final bottle is now cleaved of any blood. Which anatomical structure is the tos likely to be xesponsible for this bleeding? = A radache A. Anterior spinal artery B. Epidural artery ©. Subarachnoid vein D. Veins within the erector Spinae muse] Ve E. Vertebral venous Pplexu; s eye preload sees + £2 2s Year old woman presented in the surgical olinie with « thee-month history of dlarthoes. Ou examination she is found to have smooth 2 cm swelling in the front Pherncck Her sista: bas wothang which Way UPETaTeY Gir TWO Yeats ago, The swelling moves on swaliowing and the oarinw ierlinically euthyentd caleiton ation CYtology of the swelling shows amyloid store and serum calcitonin levels are elevates. Which of the following is the most likely diagnosis? A. Anaplastic thyroid carcinoma AB. Follicular thyroid carcinoma 4 C. Medullary thyroid carcinoma ue rn» “eh E DP. Metastatic nyroid carcinoma E pilery YJ ! carcinoma, with known prostatic hy; with acute urinary obstruction, ths risstion he, Collapses Wh a blood precy 8 ‘wpisfoviaute, What is the most kche diagno: perplasia prec i B, ehagic shock ai ©. Latex ale: tic shock asovagel syncope on 201. A 70-year-old man has a b "cell carsinotna resected frosa bi {be'most inporant prognostieindieain is tell temple. What | “A. Clinico-pathologicat type g 8. Completeness of exciéion ©. Titer ity of the Iymphoeyt yinphava Mitotic ince infiltrate, cular space invasion Ain the tumour isiands fuam has undergone e mergency surgery to rapuic a leaki abdominal exeuryom, He underwent transfusion with 6 units Pood teste were performed tw hou, pos b WRC 12.3 > 10%, pt B aortic * of packedtred cell ‘op haemoglobin 10, { edb, Haielets 40 10%, ApYT (Activated Pura! Thromb Titte) 36 seconds, INR Gntemationat Nosmmalised Ratio) 2.2, fibring q fibrin degmdation’ roducts 25 mig/mi. What wilt these resui i @) - ut atively: be consistent wis erin ntravascular coi N of antithrombin J levels uf vitamin ¥ ‘ilation t dependent factors nogen inactivation if E, Protein C deficiency 103. A 46-year-old woman ig ion bio years ag eehocarciogsam chow th ‘ber INK Guiemetiona following? Pre-admission clinic. She had a myosa ial win @xathination abe is in aia! fibrillation and 4 recent She had a dilated left ventricle. Sherig on warfarin and Notinalised Ratio) is 6.1. Warfarin inhibits whichof the Kallikrein ca moe gor. Hyponatraemia < § torr epi con ~remmataet snseetar essisincesinel 104. A 29-year-old pregnant woman is having uncoutrolle ‘ral for pain reliel Which of the following She has an labour pains. is tue? A. An epidural puncture shouid only be performéd below the L1 level to avoid dainage to the spinal cord 5. An epidural puncture should only be performed betow the L4 level to-avi Spinal cord oO nerve roots and a venous plexus The epidural space (also known as the extradural spece) contains num oid the eroUus D. The ligamentum flavum is rarely pierced during a low epidaral procedure yet %. The supraspinous and interspinous ligaments aro rately pierced during the procedure 105. A 28-year-old motoreyélist presenis to the Emergency Depasiment after being « involved in a road traffic accident, He has a closed isoldted injury to his right ieg. Radiographs show a segmental fracture of the tibia and the fibula, Intramedullary nailing of the tibia is undertaken. Six hours follow: ofsevere calf pais which doesnot respond to analgesics. Movement af the wan Severely increases the pein. What i the most likely eause of this pain? A. Deep vein throgabosis B. Deep infegtion C. Embolic ebisode 7D. Raised intha-compartmental pressure BE. Movement of the unstabilised fibula 106. A 65-year-old woman with metastatic bre: confused and acutely unwell, with naus nornai limits. What abnormality is maost likely to condition? VA. Hypercalcecria ~~ B. Hyperkalaemia a ©. Hypocalcaemia D. Hypokalaemia ~ anit st cancer and yoru is admitted to hospital ng. Her vital signs are within ontribule to her clinical sensation over t injured? 7B. Common peroneal nerve EJ ~~" 3B. Deep peroneal nerve . Saphenous nerve 3. Superficial peronee £, TET w 3 Ssyeavh WONT SoMpHAS OT wealaess of dorsiflexion of the ankle Physical examination reunals abeecr * dorsum of the foot. Which of the follovring nerves is most lileely piso nareeaiooR ema castes i . hsieuctive pulmonary disease! (COPD), nes Confitsed afler a right hemicolectomy, She is of: 30% oxygen and her 1d gas analysis shows: [BS kPa A760 | (Nom From what is she most-likely to be suffering? A. Severe asthma attack B, Emphysema C. Pulmonary embolus D. Type 1 respiratory failure 7 EE ype 2respicatory failure WYper Capes yp OK GENTE 109. A 19-year-old man fell down a flight of stairs. On presentation to the Emergency Department his Glasgow coma scale (GCS) was 14/15 with no focal neurological signs. Aa hour later he vomited and his GCS deteriorated to 10, Hé developed a fixed and dilated left pupil; Which of the following accounts for tie pupillary signs? : . . Compression of oculosiotor nerve parasympathetic fibres . Compression of sympathetic fibres to the left iris 1 . Compression of the afferent fibres in the left optic nerve Compression of the Edinger Westphal aucleus ma, presents to the outpatient clinic, He fell awhowasdly, ‘with excessive lateral flexion Ofhis neck fo the left'side. He desczibes sone paraesthesia over his right shoulder and Jateral arm, which.appeers'to have been improving over the past hone. What is the most likely injury suggested by this history? » A. Axonatmesis of the C5 herve root . B. Axonotmesis of the Tl herve ret LC. Neyropraaia of the C5 slerve root D. Netroprazie of the TY ilerve root F, Neurotimesis of the C5 nerve foot LLL A 62-year-old. woman presénts with a firm inegular mass in the unper outer quadrant of the right breast. This appears malignant on mammogiaphy, and THe B. Primary node 5. Seconda D. Sentinet node, al ! E. Virchow’s node node 112. A 50-year-old woman presents with a history of faecal incontinence over the past few years. She had a prolonged and difficult fst stage of lebour20 years = previousiy. Physical examination reveals a relatively lax anal sphincter, Which nerve is likely to have been damaged in labour? A. Autonomid nerves to the rectum B. Genitofemgral nerve €. Lumbosacrl trunk D. Obturator nerve Ve Pudendal nérve 113. A 26-year-old man presents to the Etnergency Department with extensive bleeding from his arm, after sustaining a glass injury-On examination there ig a7 em transverse laceration atvoss the anteridr aspeét of his elbow. On exploring the cubital fossa, you would expect the brachial artery to be: : i je Xo wed he eet dew aoe Brvopstw fee A. anierior to the median nerve B. latéralto the biceps tendon gor: lateral to the median nerve D. medial to the median nerve E. superficial to the bicipital aponeurosis 114, 4. 53-year-old man complains of ‘crushing’ chest pain at rest, He has chronic Lypertension and is a heavy smoker. His pulse rate is 138 bents/miaute and alood pressure 140/90 mmHg. What is the principal cause underlying his reduced coronary artery blood flow? DA. Decreased cardiac muscle wall tensio; 7 3. Decreased coronary oxygen extiactio ‘C. Decreased diastolic interval q oa D. Decreased systolic interval B, Decceased vpgal tone 15, & 38-year-cid woman, a koown patignt with thoracic right-sided meningioina, resents with fegtures suggestive of a Brown-Séqiiard sytidrome. The clinical, findings will include which of the following? 1 4, Left sided weakness, left sided proprioception and vitication loss, sight sidec Joss of pin prick sensation . UW. Right sided weakuess, right sided pruprivception and vibration loss, lefi sided Joss O1 pin prick sensauon Q: C. Right oided woakuess, sigh id prouloceyth loss of pin prick sensation D. Right sided weakness, left sided proprioception and vibration loss, right sided loss of pin piick sensation eft sided weakness, right sided proprieceptivu esi vibration loss, left sided of pin piick sensation 7” te a 70-year-old woman prestints wilh a fractured left hum = show an rclvtc lesion at the site ofthe fracture, Histological examisaites at semple taken atthe time of interna fixation shows.a metastatic tereinage Where is the most likely primary site? ; CUS. A ® Breast ~ Colon : C. Endometrium D. Ovary ; : E, Stomach 17. A 74-year-old man presents abnor: ith 2 pulsatile swelling in his abd lomten. Fhe principal ‘ality within the wall of arterial ancurysme is loss of A. adventitial coliagen B. adventitial elastin ©. intimal collagen intimal elastin’ | . ediatelosin | EY A Meyeat-old vrowen presents with a swelling in the thysoid gland. She also ips colorged lymph nodes in the tft anterior triangle of the week Whick of the follow: the mast lkely-diagnosis? po jc careinena #. Follicular adenoma Follicular carcinoma Medutlary carcinoma _7© Papillary carcinoma : M9. A20-yearokd man with|s severe head injury is being veut Positive pressure, Which of the fol A. Decreased extravettdh LAB. Decreased cardize preload 8 ©, Inowease in intrathoracic blood vol + D. Increased cardize prétoad s B. Increased cardiag 5 4 using intermittent lowing is a physioiogical conseqisense oF tis fnid volume roke volume gastric pain and peofase #0-yeav-old man presents with « one-week history of o Ukcly aucisboile wonseyieneeig: hypochioraemic acid raised wring seduced renie: release t 123 f L aad en Entereostal an A 22-yearold men anives into the fifth left intercostal space in the mi into the dreinage,hottle, Which of the followin, ute haemorrhage? y 8 Left pericardiophrenic artery C. Lingala o ‘he lung D. ‘ventricle of the heart E. Spiecn A. Second aortic a Prd —b Corbi © B. Third aortic arch We Aodid atch © : 5% Aeppene bth in the fetal cireulation, the majority of blood paging froth the inex into the righf atin s ently passes next ini which’ one of the f c y arlery L ventricle A.36-yeat-cld byfeher injures his left index finger with « knife ACRTEUOD OF WE PATTIE ISHECT tn flex the isin’ fntermtatangeal or digitorum pro Flexey digitorum 5 divis the Emergency Department with sudden breathlessness duis to « large ppeumothorax, An immediate chest drain is inserted is haemorrhage sirnctuwes is the most likely cause Vb 2nd —m Mizappear. Yo % ‘ven caya Bowing? Fe (Be) ap bachie Geph Hie wd Rr sto clavin Sr wa Lt agpumenny ot bones Te ge ete esesenecenanennanemen see { F of pnewtonia. Culture showed Psesdomonas aéruginesd. His mother confirms that li fum ieus is the neonatal period. Which of the following is the most likely finding in this boy? 122A T-yeor-old boy prevents-with recurrent a A, Increased aweat alanine transaminase increased sweat chioride c C, Increased sweat creatinine Increased sweat potassium E, Jnereased sweal urea Lic Pibros $ : H : 130. AA2-year-old woman has a cholecystectomy and develops a self-limiting postoperative wound infection. By what process would Yacterial ingestion have sn enliaaced? - : Apoptosis Antoghagy otaplasia Opsonisation 7B. Phagosytasis vos> 1 of rectal tumour a patient experiences Grectiie dysfunction. Which 131, Afer resesti ely ro have been demiged in surgery? of the fattowing nerves is most Ii moral nerve . sal pier . Pelvic splanchnic nerves 1. Perineal branch of S4 4, Pudendat nerve > exten A. Genito! sphine ber J ancl giseo es bowel moveravats presents with inary tract infection. CT seat show # mass involving the sigmoid colon and the biadder. What is the conrmonest cause of this presentation? A. Adenocarcinoma of the sigmoid colon _-B. Colonie diverticular disease Le \"G. Groha'sdiscase ~e entersen trie fa 1. Transitional cell carcinoma of the bladder TL. Ulverative colitis: 134, A 30-year-old ini is assaulted with 2 hamaner aud sustais fracture al the Yertex of the skull. Profuse venous bleeding is noted at the fracture site, Which vascular structure is most likely to heve been peiforatéd? A. Cavernous sinus : B. Inferior sagittal siaus C. Sigmoid sinus D. Superior petrosal si 1: Superior sagital sinus us 135. 57-year-old heart teanspiant recipient is keen to join th tation programme. Which of the following factors is most likely to ineréuse cardia. output in this patient during moderate exercise? A, Decreased negativ pressure 7 i B. Decreased vendus tone C, Decreased ventricular compliance 7 B- Saoreased atcial filling 0 H, Incrensed intraperieardial pressure intrathorag 2. She had u rena! sat for 536. An 18-year-old woman fell 9 metres while rock climb trausplant two years ago. She is brought to the Emergericy Dea = esinisciven, When comecily prescribed, which nf tae following abalgesic dzugs would be most likely to adverssly affect het Fema in this situation’ AL Wi B, Dihydrocedsin 9 woman bis a lange, expanding ovarian ing u lateral pelvie wall clearance of lyin i of painful spasins in the gr ial side of her thi rams. i eh i nt is found to have a sodium of bo signs of hypovolauia, Hig plasina urloe vsmolality is of thést 3s? p 70-year-old pa Above a mmol/l. On GOO inOsea/l., Which of the following is thy tl diabetes insipidus rat salt wasting syndrome 0 Nepluogenic diabetes insipidus jadrome of inappropriate anti-diuretic hormo: 2, Water deprivation ih A .suggcon is canying out an ¢ ‘ Which sty ective spleneetoat vill be divided ia orde: to mod ¥- congenital abesi osplenic ligament . esser ommentunn 6 fenovenel ligament, D, Pixenicocoli¢ ligament E. Shout gastric sels appointment, e paipeble spieen, four fingers’ breadths below tke vost ‘xd, Pull blood counts have shown a persistent thromboeylopaenin. nw examination has shown megakervacytic bypofplasia. What ss aly Gause of the thrombocvtopacnig?. : : he bone marow estruction in the liver wetion in the spleen ieu et dest is removed snwolalty 8 286 200sr/i, Bad i best explanetion (ADH) secfetion spherocytosis. ise the spfean tom the pet ‘year-old ine has established post-hepalitic cinshusis. At « follove-up iargin, is Rone he my sccaecteamrsemamnaseapamesorrsse SESE SES a RE cr [ 137, ATi-year-old man js admitted with acute urinary retention secondary te benrga prostatic hyperplasia. A urinary ‘catheter irinserted end Ke has free fluw of fe, Which of the following best accounts for the action oF the | jer mechanism that leads to the cducentration of urine in the Conceited f counter current rauitip! Loop of Henle? |A. Antive transport of solute out of thin section ascend lirmb Impermeability to water in thick ascending limb Permeability to solute in descending liawb ; peymeability to solute in thick ascending limb: a Permeability to water of thin seqtion ascending links € 1 : co | i : 138, A 52-year-old fnun is found to bave multiple niyelog h = 7 [ | appearances would be characteristic? 7 A. Diffuse thickening of the calvarium 2 air on end’ appearance: C. Mulliple frgetures UD. Multiple oseotyte lesions . FE. Multiple osteoscleratic lesions 45-year-old man undergoes resection of bis ih ondary to = Spnich of the following, effects is most Tikely to be sefn in this be ostoperatively? fh, Decreased bile salt synthesis by the yer creased concentration of bile salis in colonic co ar volume B. 1 tents | ©) Decreased mean corpuscut | L-7 D. Decreased reabsorption of vitamin BIZ E. Decreased stool frequency jay), A previously healthy 77> ==oldveman undergoes en r .pinre of the spleen. A left subelavi ne to guide fluid therapy is mserted. At i ‘with a blood pressure of 75. ye is 35.5°C. Wh fan CenREF dis { cv and acutely dyspnoeic, breath sounds on the left. Her temparaiur st this scendttio! 40 mmllg. 3g the most likely cause During the second (prolifecati und healing the predominant eells in the wound site are fibroblasts. This cell is of mesonchysaal orig'n matrix aid collagen iéeded:to'strengthen the sear. Czoss linkage éfsollagen tequires hydroxy-proline and hydroxy-lysine tesidues, which requites a specific vitamin to be available in sufficient quantities, Deficiency of whish vitamin results in coliagen that is unstable? | A. Vitamin B2 (riboflaviney’ B. Vitamin BG fpyridoxine) L-— ©, Vitamin C (ascorbic acid) D. Vitamin D {cholecaleiferal) B. Vitamin E (tocopherd) (42. A 19-year-oid man fail down a flight of stairs, Cn presentation to the Emergency "Department his Glasgow Coma Score (CICS) was 14/15 with no focal neurological An hour later he Vomited and his GCS deteriorated to 10. He developed a ‘xed and dilated lett p pupil. Which of the following is likely fo bé seen oa the scan? 1-7 B, Leftssided extradural haematoma - v C. Lef-sided midbrain hematoma Ovy D A. Left-sided acule subdural haematoma —* ea i Z D. Right-sided'acute subdural lacmatoma, E. Rightsided extradurgl baematomn 143, A 70-year-old woman weighing 50 kg undergoes open cholecysiectomy. Postoperative pain relief is administered by intravenous morphine infusion (morphine 1 mg; 5 minute lock out).“Tweaty-four hours postopéraltively she ig in an agitated, confused slate. Her temperate is 22.1°C, pnise 90 eashininute 1080 (g. Her urine output is 35 ma/bour, Arteria! blood gases Base exces ate inductd hypoventilation C. Pain induced hyperventilation i nbolus at rayocardial infarction SUSU arin eS using home is brought to the Em ment with abidominal pain and vorniting. On exaniination, she is del abdomen is distended. There is «3 qa x 4 em swelling ia the right groin tender, and there is no cougis impulse. At operation, a femoral hernie re of the following lies immediately Lateral to the hernial neck? arated 7 A. Femoral eitery B. Femoral nerve ©. Femoral vein. D, Inguinal ligament E. Pectineal ligament 145. A 64-year-old woroan is reviewed in the Emergency Department wi right groin sweiline, below and lateral to the pubic tubeisie. Which of the following forms the ntedia} boundary of the zing through which this hemin is . protruding? A. Conjoint tendon. B. laferior epigastric axtery C. Inguinal Ligament Lecuaar ligament E, Pectineel ligament 146, AB old has. had panculitis for the last five years, Which one of these: compiications is he least likely to davelop? [ A. Arthritis, B. Cholangitis {pee kitis (~. Polyacteritis , Toxic megucolon~ 147, An 82-year-old woman had @ hemi-archroplasty of the hip six hours 2g0. Sue isan insulin-dependent diabetic and has bees on long-term: non-steroidal analgesics. Ip. the last three hours she passed, via her urinary catheter, a total of 20 ml of urine, ‘ Her pulse rate is 120 beats/minute and regular, her blood pressure 100/60 mmFip and capillary refills 3 seconds, Biood urea and electrolyte leveis are: (Eee So. Creatinine saa | pining |= 13> mmmowe Ome | Mo amal 17 iat Q@sagy 135 otassi m The most likely cause of this is \ a4] a. acute tubular piwopat fal antin’ fits rosis toryvoiume in Re piratory lest paramecer noed vila! capacity} is the best pre ot for mosbidity and ercoud; FYE =F mortality after surgery? A. Contionted flove volume loop 1 2. Aptasti io virus iuieation is une spontgaee ny a at-gld women presents with guaelrant of the tiglt breast. ‘This is Cine nedle aspiration cytology. She is treated with clearance, Whied of tie folly AG. A, Absence of HER-2 amplification 1B, Bloom and Richardson Grade 1 . More than three positive axillary no rogent receptors of the ring finger is corvect? * (LEXA, Hes a cutmeous innervation from the susiat nerve 3. Has no cutanedus iunesvation from the median w C. Has ue cutansous intervation from the.wins: ‘Hag the flexor digitorura profiindus inserting silmar interosgoous muscle | ¢ : . & ee eur-ald za piesents with deep vain thrombosis iI" ATS ing histopathological fing y wt have surgery ths Dupuytren ering finger of the righthaud. Which ow ofthe following regarding tia snalemy frm irregular mass inithe upper onter -vn (o be raalignanl on mammography local excision and azillary ings would imply a better Sontrreture: to selease ao. ¢ phalanx _»5 Add —» Pag 5» Ab? Dal viskd tomogeaphy 5. Borat conlitins the presence of a throrabus in Hie common iliac Yein extending into the inferior vena caves The common iliac veins jois to form thie inferior, vena cava wt which vertebral level? nee ee x 562, A 35-year-old snotorsyclist is suyolved ine road sali temporal bone, completely (ransecting the facial nerve: ganglion. Which function, of this nerve remains? La Lacrimation AQF" B. Sasivation Scum the submandibular gland Sensation to the anterior tongoc —¢ Latwxfout o D. Smile Taste -year-old motoreyclist is brought to the Emergency Departmént after a road us © 163. A30- aj open fiacture of the right femur. On artival, heis trafiie aecilent. He fi tachypnoeic and’ conta, with cold atid clammy skin, Which of fe following FO, ~ physiological cizanges is Mnost likely to be seen? : eae my A. Decreased reabsorption ef water from. the collecting tubules J we B. Decteased serum bicarbonate level " Increased pH of blood : [ 7} 1D. Increased secretion of sodivm in the urine E. Increased synthesis of glycogen in the fiver 164. A 50-year-old man on the waiting list for a hip replacement attends the pre- f se Gperative clinic. He has a body mass index (BMA) of 41. and o rik of pest-operative { hypoxia. Reduction of which of the following volumes or capacities is tis risk st likely to be due to? : | A, Expitatory reserve volume B. Functional residuat capacity Inspizatory réserve volume : Oo . Total Jung. volume _ Vital capacity | 165, A d-year-old boy presents to the Emergency Deptartment with a two-day history of | headache, vomiting and drowsiness. A. CT scan reveals dilatation of both his lateral ‘yeutsigles dnd his third cerebral ventricle. His fourth ventricle vas of normal size It is suspected that he has an obstruction to his cerebrospinal {uid flow. At which the following sites is the obstruction most likely to be? A, Cerebral aqueduct (of Syivius) 55) nfiniibulat recess : C. interventrieslar foramen (of Moti) 1, Lateral foranten of fourth ventricle (foramen of Lusch); E, Median foramen of fourth ventricle (foramen of Magenidic) 43 mee C3 jc ome Fo Ee eet UREN een vinucly, 1s 0 plasiia-sodiu 9 is 322 mamol/L. inspection wnitten up for Fourhourly intravenous 5¥% fluvose infusions. What the most Tikely cause for the hyponatraemia? failure, 7 has bea: ic hormone) response to surgery e 7 glucose infusions rome of ineppiopriate antidiuretic ROAMORe Use of loop diuretic in the long term During arch aoitography, the vertebral artery would be scen tolarise directly from which of the following? H A B. . Arch of the aorta Brachiocgphali¢ artery Common carotid arlery D. Iniemal carotid.artery 1B. Subclavian artery : 168. A 60-year-old non-smoker presenta with a three-month historyjof loss of weight, rnaiaise and breathlessness. On examination, he hes left supraciavicular iynoph node enlergerient} Chest radiogreph reveals multiple bilateral gpacities, What wii) biopsy of felt supra-clavieu ar lympb node'most probably show? onic inflammatory cells. : Grenuloma : €. Langerhan’s type giant cells \-8r Multiple ebuormal mitotic figures ia celis E, Reed Stenberg cells wp 20 168. A 12-year-old child was admitted to the Emergency Departmesst having inbaled a Peanal, Where vag dhe peanut most Ikely to bave lodged? lower tob$ bronchus A. Left ypyver vtng boone {. Rigi lower tobe bronchus. : 1D. Right midute tobe bronebus 2-77 ye bronchus Right up 44 | | i cojel mie with Dar ie vespuni howto: SAL defy ose B. Faresympatls Pulinonary vaneunay tes rere yympathetié activity) * wate i aipple. Matumography is nalyelptal duets, What is the moat Hkely patholog: cystic disease 2. Phylloides tambue ,-Plasmecell mastitis A SG-yearold on pres Mit Just (wo weeks. On es 4 ition mous. Which juce naible fi -year-gle multiparavs woman whe sunske 2 Pein tender himp inher right bre; nel tgs Welt Tecate: aed ve gceupying leston in she right parietal Jobe, Te develops a feminuopia, Which part ofthis vi W : BeOptic chins : Fue. Optic radiation 1, Retina 9 IB. Second era nerve: solacisution is shown by w Tikely to be wife treatment with Which gact of th {HDU), te Ye, Aoi a Seninhysts Ap Appoint} Apoin ct em lat sping a ad the pubis Bo. J seule. esaeintt vera ioxin, his AP slows ated beds re nVengus wave form day: Mer auserior resection. He has develog hand hypotension. A. point midway bets midway berween the grenter trochanter and the pubic symphysis cn wnferior and # Binergoncy Deemer xlomian! pair ad and her blood sugargs 42 mero. The absent? ‘Which of the fol’ owing fandmates woutel be the most terial blood gas bangle? seen the entesior superior iliac spine and the p veen the-anterinr superior ino spine au the F cin lateral is the pubic tubercle : idwuy point between thd anterior superior fifad syiniphysis Js the is been treated a ss for 4 chest infection. The éreatinine fs 406 nolL rnost likely sevare electrolyte atusornality is :

You might also like